OMM Semester 1 Savarese Practice Comlex Exam

अब Quizwiz के साथ अपने होमवर्क और परीक्षाओं को एस करें!

You are assessing a newborn in the hospital nursery. The child was born yesterday, delivered at 41 weeks by spontaneous vaginal delivery complicated by shoulder dystocia. At birth the child was 10lbs 2oz & measured 21 inches from head to toe. The Apgars were 9 & 10. The mother has been attempting to breast feed, however the child is having difficulty latching on to the breast. On examination, the infant appears to be nervous & irritable. Flexing & abducting the hips while exerting pressure on the greater trochanters demonstrates that the child has a R hip click. Which of the following best explains the most appropriate course of action for this Pt's hip click? A. Weekly osteopathic manipulation to decrease myofascial restrictions & optimize femoral head position in the acetabulum B. Monthly radiographs to determine the position of the femur in the acetabulum C. Monthly ultrasounds of the hip to determine the position of the femoral head in the acetabulum D. Pavlik harness w/ follow up CT scan to determine the adequacy of reduction & position E. Rigid casting w/ the hips in frog leg position

Correct: Answer D: The Pt has a positive Ortolani's sign. This is indicative of congenital dysplasia of the hip. Tx for this involves application of a fundamental osteopathic precept: growth & maturation of the skeleton follows function. Many orthopedists today use the Pavlik harness to hold a dislocatable hip in place. This brace allows the child to flex the hip & kick while the hip is in a physiological position. Answer A: Osteopathic Tx may be used in addition to the above Tx. However, alone it is not considered to be the most appropriate course of action. Answer B & C: Monthly radiological testing & observation is not an adequate course of action. As the child gets older, especially as the child begins to ambulate, it becomes more difficult to obtain adequate reduction w/out surgery. Answer E: Rigid casting is an older form of Tx. Placing the hips in the "frog" position may put added stress on the hip while it is reduced & is generally considered to be the optimal position.

An 81 YO male w/ atherosclerotic heart disease was admitted to the hospital w/ an exacerbation of COPD. He eventually developed pneumonia & went into acute respiratory failure. He was intubated & received assisted mechanical ventilation for 2 weeks. He was extubated & transferred to a general medical floor where he spent 2 additional weeks. Today he arrives at a skilled nursing facility where you are the covering house physician. On your admission Hx & PE, the Pt complains of low back pain. On examination, you notice that this severely debilitated male has generalized muscle atrophy & requires maximal assistance w/ bed mobility. He is unable to tolerate sitting or standing. You conclude that his back pain is due to his prolonged bedridden state. Which of the following Tx protocols is best suited for this Pt? A. Low velocity moderate amplitide techniques followed by gentle isometric exercises to strengthen the L-Spine. B. Myofascial release of the sacrum followed by isotonic exercises w/ low resistance to strenghten the L-Spine. C. Rib raising to help w/ respiration followed by ME to the pelvis & lumbar SDs D. Indirect Tx's to the lumbar fascia followed by direct gentle articulatory techniques. E. Myofascial release to the thoraco-lumbar junction followed by gentle HVLA Tx

Correct: Answer D: The elderly Pt is recovering from severe pulmonary complications from his COPD. He has been bedridden for several weeks & he has coronary disease. As noted in the Foundations of Osteopathic Medicine elderly Pts & hospitalized Pt's typically respond better w/ indirect techniques or gentle direct techniques, such as rib raising. Answer A: This Pt could not tolerate isometric exercise due to his low vitality. Answer B: The Pt will not be able to tolerate isotonic exercises w/ low resistance, even though myofascial release of the sacrum may help this Pt. Answer C: Although rib raising will help the Pt's respiratory status, this Pt will not be able to tolerate ME because of his low vitality. Answer E: Although this Pt may tolerate myofascial release, he has been bedridden for several weeks. Thus, his bone mineral density may be decreased to the level of osteoporosis. HVLA techniques are thus contraindicated.

A 53 YO male comes to the ED complaining of low back pain. He states that the pain he is experiencing now is similar to his back pain that he had when he herniated a disc 2 years ago. He brings in X-rays & an MRI from 2 years ago. X-rays show spondylosis at L5/S1. The MRI demomstrated a L posterior lateral herniated disc at this level. The back pain radiates from the L-Spine into his L foot. Which of the following organs would be least affected by an autonomic imbalance of the upper L-Spine? A. Ureters B. Urinary bladder C. Descending colon D. Penis E. Adrenal medulla

Correct: Answer E: Adrenal medulla. The adrenal medulla receives sympathetic innervation from T10. Answer A: The ureters receive sympathetic innervation from T10-L2 Answer B: The urinary bladder receives sympathetic innervation from T11-L2 Answer C: The descending colon receives sympathetic innervation from T12-L2 Answer D: The penis receives sympathetic innervation from T11-L2

A Pt complains of CP, especially w hen she exhales. The pain is localized to ribs 8-10 on the L. Evaluation of ribs 5 - 7 on the R reveal that the ribs are depressed anteriorly & the inferior edges of the posterior rib angles are more prominent. The ribs move caudaly during exhalation but do not move cephalad during inhalation. What is the most likely Dx? A. Ribs 5 - 7 inhalation dysfunction, pump handle predominant B. Ribs 5- 7 inhalation dysfunction, bucket handle predominant C. Ribs 5 - 7 inhalation dysfunction, caliper motion predominant D. Ribs 5 - 7 exhalation dysfunction, bucket handle predominant E. Ribs 5 - 7 exhalation dysfunction, pump hande predominant

Correct: Answer E: In exhalation dysfunctions, the rib moves caudad on expiration & is restricted on inspiration. In pump handle dysfunctions the findings are concentrated on the anterior part of the rib in the mid-clavicular line & the rib angle. Answer A, B & C: In inhalation dysfunctions, the rib moves cephalad on inspiration & it is restricted on expiration. Answer B & D: In bucket handle dysfunctions the findings are concentrated on the lateral part of the rib in the mid-axillary line

Match the description with the best term: A Pt w/ a L-sided paresis from a stroke ambulates by leaning to the R side, advancing the L leg in a circumduction pattern while the L foot drags on the floor. A. High steppage gait B. Ataxic gait C. Shuffling (festinating) gait D. Waddling gait E. Hemiplegic gait F. Elevated pelvis gait G. Antalgic gait H. Gluteus medius gait I. Scissor gait

Correct: Answer E: In this type of gait, the leg is extended at the knee w/ plantar flexion at the ankle. The Pt leans to the stronger side (in this case the R side) & advances the leg by curcumducting it. Since the ankle is plantar flexed the foot tends to drag against the floor.

Which of the following best explains the role of the quadratus lumborum? A. Inhalation causes relaxation of the quadratus lumborum & permits cephalad movement of rib 12. B. Contraction will result in posterior translation of ribs 11 & 12. C. It is the 1st structure to react to stress in the lumbosacral area D. Contraction will increase lumbar lordosis & increase the intercostal space between the 11th & 12th ribs. E. It plays little role in lymphatic drainage.

Correct: Answer D: Contraction of the quadratus will extend, & sidebend the L-Spine as well as pull rib 12 down (caudad) thus increasing the 11th & 12th intercostal space. Answer A: Inhalation results in eccentric contraction of the quadratus lumborum resulting in cephalad movement of rib 12. The latissimus dorsi also plays an important role in the movement of rib 12. Answer B: The quadratus lumborum does not attach to rib 11. Answer C: Although the quadratus lumborum tenderpoints have been referred to as the most often overlooked cause of low back pain, it is though that the iliolumbar ligament is one of the 1st structures to react to stress in the lumbosacral spine. Answer E: Tx of the diaphragm as well as the quadratus lumborum has proven successful in helping to promote lymphatic drainage.

Manipulation of which spinal segments may decrease BP by decreasing fluid retention? A. C3-C4 B. C5-C7 C. T5-T9 D. T10-T11 E. T12-L1

Correct: Answer D: Experiments show that hypersympathetic tone to the kidneys for several weeks causes renal retention of fluid resulting in chronically elevated arterial pressure. Therefore, Tx toward the kidneys (T10-T11) may decrease fluid retention & decrease arterial pressure. Answer A & B: Manipulation of cervical segments will not decrease hypersympathetic tone in the kidneys. Answer D: T5-T9 receives sympathetic innervation from the upper GI tract Answer E: Manipulation of T12-L1 will not decrease hypersympathetic tone in the kidneys. It may alter the sympathetic tone to the ureters, however this is not the problem.

Abnormal visceral afferent input from cardiac muscle is most likely to manifest somatic changes in paraspinal musculature adjacent to which vertebral level? A. C5-C7 B. T3-T4 C. T7-T9 D. T9-T12 E. L1-L3

Correct: Answer B: Abnormal visceral stimuli from cardiac autonomic afferents are most likely to cause paraspinal musculature changes at T1-T5. Answer A: Although the heart does receive fibers from the cervical chain ganglia, abnormal visceral input is less likely to produce C5-C7 paraspinal changes. Musculoskeletal changes due to vagal hypertonicity can be seen at C2 & the cranial base. Answer C,D & E: Paraspinal musculoskeletal changes from T7-L2 typically will occur from visceral dysfunction below the diaphragm.

A 34 YO female comes into your office complaining of mild L-sided thoracic pain. The pain started about a week after the Pt began driving her new sports car that has very low riding seats. The pain is non-radiating & worsens w/ inhalation. X-Rays & EKG reveal no abnormalities. Your structural exam reveals that ribs 3 -5 on the L are more caudad & lag behind during inhalation. Tenderpoints are noted on ribs 3 - 5 in the L mid-axillary line. Which of the following correctly describes counterstrain for the above dysfunction? A. Minimal thoracic flexion, L rotation, R sidebending B. Minimal thoracic flexion, L rotation, L sidebending C. Marked thoracic flexion, L rotation, L sidebending D. Neutral thoracic spine, R rotation, L sidebending E. Neutral thoracic spine, L rotation, R sidebending

Correct: Answer B: The anterior tenderpoints for ribs 3-6 are all treated by sidebending & rotating toward the side of the tenderpoint & encouraging slight flexion. As a general rule, rib tenderpoints are treated for 2 minutes as opposed to 90 seconds. Dr. Jones felt that positioning for rib dysfunctions could be uncomfortable for the Pt & an extra 30 seconds were needed for the Pt to relax.

A 34 YO male is complaining of severe R-sided low back pain radiating into the foot. The pain started yesterday & is associated w/ calf-cramping. He describes a shooting pain into his R-foot that is made worse w/ forward flexion of the L-Spine. On examination, the pelvis is shifted to the L. There is an absence of lumbar lordosis. The L-Spine is Sidebent to the R. The sacrum has Rotated right around a T oblique axis. There is a positive straight leg raising test. After 2 weeks of conservative management, the Pt comes to your office still having significant pain. Electromyography shows abnormal findings in the anterior tibialis & extensor hallucis longus. Based on the innervation of the muscles involved, which nerve root is most likely affected? A. L3 B. L5 C. S1 D. S2 E. There is not enough info given to specify an answer

Correct: Answer B: The anterior tibialis is innervated by the L4 & L5 nerve roots. The extensor hallucis longus is innervated by the L5 & S1 nerve roots. L5 is the only nerve root listed above that shared these 2 muscles.

During evaluation of a Pt's upper back pain, you notice that your R thumb is more posterior upon palpation of the TP of T4. T4 returns to neutral position w/ flexion of the T-Spine, whereas extension of the T-Spine increases the asymmetry. If treating this t w/ HVLA in a supine position, which of the following is correct regarding hand placement & direction of force? A. The thenar eminence is placed under the R TP of T5, force if perpendicular to the table B. The thenar eminence is placed under the R TP of T4, force is perpendicular to the table. C. The thenar eminence is placed under the R TP of T3, force is 45 degrees, caudad D. The thenar eminence is placed under the R TP of T4, forece is 45 degreed cephalad E. The thenar eminence is placed under the R TP of T5, force is 45 degrees cephalad

Correct: Answer B: The thenar eminence is placed under the R TP of T4, force is directed perpendicular to the table. Using the Kirksville Krunce technique, to reverse all planes of this particular SD, the thenar eminence is placed under the R TP of T4, to induce L rotation of the spine. The downward projection of the SP at T4 will induce extension at this joint when HVLA is applied. Answer A: When treating a flexed SD the thrust is directed at the dysfunctional segment, not the one below. Answer C:Treating T3 will not have the greates effect on the SD at T4.

A 34 YO female complains of dyspnea, fatigue, weakness, weight loss & increased appetite. On PE you note a goiter, warm sweaty skin, & a hand tremor. You would expect to find tissue texture changes due to a viscerosomatic reflex at which spinal level? A. C3 - C5 B. T1 - T4 C. T5 - T9 D. T9 - T12 E. L1 - L2

Correct: Answer B: This Pt has signs & symptoms consistent w/ hyperthyroidism. T1 - T4 receives sympathetic innervation from the thyroid. Answer A: Although C3 - C5 may have tissue texture changes from local effects, it is less likely to be involved in a viscerosomatic reflex since the sympathetic nervous system stems from T1 - L2 Answer C: T5 - T9 receives sympathetic innervation from the upper GI tract Answer D & E: T9 - T12 & L1 - L2 receive sympathetic innervation from below the diaphragm.

A 33 YO female comes to your office w/ a 3 day Hx of severe HA. The pain originates from the base of the occiput & radiates into the R temporal region. The Pt reports that she started experiencing nausea, vomiting & vertigo today. Pain intensifies in brightly lit rooms. She smokes 1 pack/day. Her allergies include ragweed. Osteopathic structural examination reveals tenderness in the R trapezius & R temporalis muscles. Palpation of these tender areas does not refer pain. There is a tenderpoint at the articular pillar of C2 on the R. The Pt returns to your office for follow-up in 3 days. She reports that her HA has resolved & she was able to get some rest after your OMT TX. Which of the following best indicates follow-up care? A. Since symptoms have resolved, there is no reason to examine the Pt & no need for further OMT. B. RE0evaluate your Pt today & determine if further intervention is needed. C. Repeat the same OMT Tx today & have the Pt return in 1 week if her symptoms return D. Repeat the same OMT Tx 2 more times to insure adequate resolution of SD E. Refer to PT for initiating ultrasound therapy to the trapezius.

Correct: Answer B: This Pt most likely has spondylolisthesis. This is not an uncommon cause of back pain for young athletes; especially those athletes that are involved w/ extension based athletic activities, such as lacrosse players. Pain is usually localized to the lumbo-sacral spine. Neurological examination is normal. There may be a positive "step off sign." In this case the spinous process of L5 is prominent, whereas the SPs of the remainder of the L-Spine are not. Answer A: Since the neuroligical examination is normal, the Pt is not likely to have a herniated disc. Answer C & D: Osteoarthritis of the L-Spine is normal, this Pt is not likely to have a herniated disc. Answer E: Although facet tropism is the most common anatomic abnormality of the L-Spine & can lead to degenerative changes in the L-Spine, alone it is not primary cause of low back pain in the above Pt.

During evaluation of a Pt's upper back pain, you notice that your R thumb is more posterior upon palpation of the TP of T4. T4 returns to neutral position w/ flexion of the T-Spine, whereas extension of the T-Spine increases the asymmetry. Which of the following correctly describes the Tx position when treating the above SD w/ facilitated positional release? A. Marked extension of the T-Spine, R rotation, R sidending B. Marked flexion of the T-Spine, R rotation, R sidebending C. Flattened thoracic kyphosis, R rotation, R sidebending D. Marked flexion of the T-Spine, L rotation, L sidebending E. Marked extension of the T-Spine, L rotation, L sidebending

Correct: Answer C: Although facilitated positional release is an indirect technique, SD of the spine are treated w/ the spine in the neutral position. This entails flattening the thoracic kyphosis, & flattening the lordosis of the L & C -Spine. The segment is then positioned away from the restrictive barrier. In this case, Sidebent & Rotated right.

A Pt tripped & fell backward on an outstretched R hand. The Pt complains of pain on the lateral side of his R elbow. The PE reveals the R forearm prefers supination & the radial head is tender to palpation & restricted in posterior glide. The most likely Dx is: A. Lateral epicondylitis B. Posterior radial head C. Anterior radial head D. Adduction dysfunction of the ulna E. Abduction dysfunction of the ulna

Correct: Answer C: An anterior radial head dysfunction would cause the forearam to prefere supination & the radial head would resist posterior glide. Falling backward on a supinated forearm can cause this dysfunction. Answer A: Lateral epicondylitis is a strain of the extensor muscles of the forearm near the lateral epicondyle. It is usually due to overuse of the wrist extensors & not typically associated w/ trauma. Answer B: A posterior radial head dysfunction would cause the forearm to prefer pronation, & the radial head would resist anterior glide. Falling on a pronated forearm causes this dysfunction. Answer D: An adducted dysfunction of the ulna would have a decreased carrying angle, the hand & wrist would be adducted, & the olecranon process would prefer lateral glide. Answer E: An abducted ulna causes a decreased carrying angle of the arm; the olecranon process owuld be restricted in medial glide, & the wrist & hand would be abducted.

Match the description with the best term: A 63 YO male w/ a R peroneal compression neuropathy has a complete foot drop. When he ambulates he flexes his R hip more than his L in order to clear his R toes. A. High steppage gait B. Ataxic gait C. Shuffling (festinating) gait D. Waddling gait E. Hemiplegic gait F. Elevated pelvis gait G. Antalgic gait H. Gluteus medius gait I. Scissor gait

Correct: Answer A: A high steppage gait will typically result from a foot drop. In this case the effected side is raised higher (hip is flexed more) in order to clear the foot.

A 33 YO female comes to your office w/ a 3 day Hx of severe HA. The pain originates from the base of the occiput & radiates into the R temporal region. The Pt reports that she started experiencing nausea, vomiting & vertigo today. Pain intensifies in brightly lit rooms. She smokes 1 pack/day. Her allergies include ragweed. Osteopathic structural examination reveals tenderness in the R trapezius & R temporalis muscles. Palpation of these tender areas does not refer pain. There is a tenderpoint at the articular pillar of C2 on the R. Which of the following is true regarding her HA? A. It is likely due to an alteration of the intracranial blood supply. B. It is likely due to hypertonicity of the trapezius muscle C. Tx may consist of myofascial stretch using vapocoolant spray D. It is likely due to degeneration of the joints of Luschka in the lower C-Spine.

Correct: Answer A: Classic migraines are thought to be caused by a vasoconstriction & vasodilation of the blood supply to the cranium. Symptoms of a classic migraine includes photophobia. Nausea & vomiting are not uncommon. These symptoms can be preceded by an aura. Answer B: Tension type HA's can be due to tension in the upper back or C-Spine muscalature. Tension HA's are not usually associated w/ nausea & vomiting. Pain is often worse at the end of a workday due to muscle fatigur. Answer C: Trigger points are taut myofascial bands that refer pain when compressed. The can be treated w/ "spray & stretch" techniques to reduce sympathetic tone to the head & neck structures to limit vasoconstriction. Answer E: Degeneration of the joints of Luschka will likely result in axial cervical pain, or if there is compression of a nerve root, pain will likely radiate into the upper extremity.

A 53 YO male comes to the ED complaining of low back pain. He states that the pain he is experiencing now is similar to his back pain that he had when he herniated a disc 2 years ago. He brings in X-rays & an MRI from 2 years ago. X-rays show spondylosis at L5/S1. The MRI demomstrated a L posterior lateral herniated disc at this level. The back pain radiates from the L-Spine into his L foot. Which of the following physical signs may be observed in the above Pt if there was a compression of the S1 nerve root? A. Decreased ability for the Pt walk on his toes B. Decreased ability for the Pt to walk on his heels C. Absent patellar tendon reflex D. Decreased sensation on the dorsum of the foot E. Sustained clonus w/ ankle dorsiflexion

Correct: Answer A: Decreased ability of the Pt to walk on their toes. Although the S1 nerve root innervates several parts of the lower extremity, it is intimately involved in plantar flexion of the foot. Answer B: Heel walking tests the strength of the main ankle dorsiflexor, which is the anterior tibialis. This muscle is innervated by L4 & L5 nerve roots. Answer C: An absent patella reflex may indicate pathology of the L4 nerve root Answer D: Sensation at the dorsum of the foot is supplied by L5. Answer E: Sustained anklle clonus is associated w/ an upper motor neuron lesion; a herniated disc at L5-S1 would result in nerve root compression.

C3 is extended, Sidebent left & Rotated left. If you decide to use a direct ME technique to correct this dysfunction how would you position C3? A. In flexion, Sidebent right, Rotated right B. In flexion, Sidebent left, Rotated right C. In extension, Sidebent left, Rotated left D. In extension, Sidebent right, Rotated right E. In extension, Sidebent left, Rotated right

Correct: Answer A: Direct ME Tx would reverse all 3 planes of C3 (i.e. C3 would be placed in such a way that the segment is against its restrictive barrier in all 3 planes). In order to correctly perform typical ME the segment must be flexed, Sidebent right, & Rotated right. Answer B: Flexing, Sidebending left & Rotating right would not reverse all 3 planes of motion. Answer C: Extending, Sidebending left, & Rotating left will place C3 away from the restrictive barrier. This position would be used for an indirect Tx. Answer D & E: Extending would not place the segment against its restrictive barrier.

A Pt complains of bloody diarrhea. A colonoscopy reveals the mucosa at the recto-sigmoid region is edematous, friable, & w/ numerous erosions. The Pt would have a predictable reflex gangliform contraction located at: A. R proximal area of the iliotibial tract B. R distal area of the iliotibial tract C. L proximal area of the iliotibial tract D. L proximal area of the iliotibial tract E. The superior ramio of the pubis

Correct: Answer C: The term "gangliform contraction" is the original term Frank Chapman used for his reflex points. It was not until much later that these "gangliform contractions" became "Chapman's points." The Chapman's points for the recto-sigmoid colon is located on the L proximal femur at the greater trochanter in the iliotibial tract. Answer A: The Chapman's point for the iliocecal area is located on the R proximal femur at the greater trochanter w/in the iliotibial tract. Answer B: The Chapman's point for the R half of the transverse colon is located on the distal femur in the iliotibial tract. Answer D: The Chapman's point for the L half of the transverse colon is located on the L distal femur in the iliotibial tract.

A Pt comes to the ED complaining of wrist & elbow pain. Yesterday while at work, he fell on his outstretched arm hyperextending his wrist. There is no Hx of direct trauma to the elbow. The wrist is warm, slightly edematous & there is significantly decreased ROM of the wrist & forearm. Palpatory exam reveals severe pain on the volar aspect of the wrist at the ulna-carpal junction. Pulses are normal & sensation is intact. X-Rays of the wrist show some soft-tissue swelling surrounding the wrist but there is no evidence of fracture or carpal dislocation. X-Rays of the elbow are negative. Given the mechanism of injury, if the Pt suffered a radial head dysfunction, which of the following findings would you most likely expect to observe on PE? A. Decreased supination of the affected forearm B. Increased carrying angle of the affected elbow C. Increased adduction of the affected wrist D. Increased abduction of the affected wrist E. Decreased posterior glide of the radial head at the affected elbow

Correct: Answer A: Falling forward on an outstretched (pronated) arm typically causes a posterior radial head SD. This will result in decreased supination of the forearm, pain at the radial head & decreased (or restricted) anterior glide of the radial head at the elbow. Answer B & C: Increased carrying angle & increased wrist adduction is associated w/ an abducted ulnar SD. Answer D. Increased wrist abduction is associated w/ an adducted ulnar SD. Answer E: Decreased posterior glide of the radial head is associated w/ anterior radial head SD. Falling backward on an outstretched (supinated) arm typically causes an anterior radial head SD.

A Pt complains of CP, especially when she exhales. The pain is localized to ribs 8-10 on the L. In this Pt, which of the following choices is the most appropriate regarding Tx for the SD of ribs 5 - 7 using ME? A. Treat rib 5 using pectoralis minor muscle B. Treat rib 5 using the serratus anterior muslce C. Treat rib 6 using the pectoralis minor muscle D. Treat rib 7 using the serratus anterior muscle E. Treat rib 7 using the pectoralis minor muscle

Correct: Answer A: In a group exhalation dysfunction the key rib is the top most rib of the group, which is rib 5. The correct muscle to use for a ME Tx of rib 5 is the pectoralis minor muscle. Answer B: Rib 5 is the key rib, but it is not treated w/ the serratus anterior muscle Answer C, D & E: Ribs 6 & 7 are not the key ribs, therefore these answers can be easily eliminated

In a Pt w/ neck pain, the 2nd cervical segment is found to resist translation to the L w/ the head in the flexed position. The segment translates equally to the L & R in the extended position. Given the above DX, which is the correct ME (direct) technique? A. C2 should be placed in flexion, right sidebending & right rotation & the Pt would be asked to rotate his head to the L against resistance. B. C2 should be placed in flexion, left sidebending & left rotation & the Pt would be asked to rotate his head to the R against resistance. C. C2 should be placed in extension, right sidebending & right rotation & the Pt would be asked to rotate his head to the L against resistance. D. C2 should be placed in extension, left sidebending & left rotation & the Pt would be asked to rotate his head to the R against resistance E. C2 should be placed in extension, left sidebending & left rotation & the Pt would be asked to rotate his head to the L against resistance

Correct: Answer A: In the above question, C2 resists translation to the L w/ the head in the flexed position. L translation induces R sidebending. Therefore, the C-Spine resists R sidebending. If it resists R sidebending then it must be sidebent L. Due to facet orientation, C2 always sidebends & rotates to the same side (type II mechanics). Therefore, if it is sidebent L, it must be rotated left. This resistance is worse when the head is in the flexed position, & eases w/ the head in the extended position. Since you name the dysfunction for where the segment wants to go C2 must be extended, sidebent L & rotated L. The correct ME technique calls for a reversal of all planes of motion, therefore the segment should be placed in flexion, R rotation, R sidebending. And, the Pt would then be asked to rotate his head to the L.

During your eval of a Pt complaining of neck pain you find a decrease in flexion & extension. What structure typically has the greatest movement in flexion & extension? A. Occipital condyles B. Atlanto-axial joints C. C3 D. C5 E. C7

Correct: Answer A: Most of the flexion & extension at the C-Spine stems from the occipito-atlantal (OA) joint. Specifically, it is the movement of the occipital condyles on the atlas. Approximately 50% of all the flexion & extension in the C-Spine stems from this joint. Answer B: The atlanto-axial (AA) joint's primary motion is rotation. Answer C, D & E: The lower cervical division C2-C7 has varying degrees of all directions, however there is less flexion & extension at these joints compared to the OA joint.

A Pt complains of hip pain for 1 day. ROM testing reveals: External rotation = Left 60°, Right 60° Internal rotation = Left 20°, Right 35°. The most likely Dx is? A. Tight L external rotators B. Tight L internal rotators C. Lax L external rotators D. Tight R internal rotators E. Lax R internal rotators

Correct: Answer A: Normal values for ROM varies due to age, gender & flexibility. Therefore it is more important to focus on asymmetry. The Pt has asymmetry of her internal rotators. The Pt could have a problem on the R or L, however, out of the answers given the only one that is possible is tight L external rotators. Tight L external rotators would decrease L internal rotation. Another possible answer is lax R external rotators. Lax R external rotators will increase R internal rotation, however this choice is not given. Answer B: Tight L internal rotators would decrease L external rotation Answer C: Lax L external rotators would increase L internal rotation Answer D: Tight R internal rotators will decrease R external rotation Answer E: Lax R internal rotators will increase R external rotation

A 17 YO female complains of anterior knee pain that gets worse when she runs, jumps or climbs stairs. PE reveals pain is reproducible on squatting & when the knee is flexed. There is point tenderness on the undersurface of the patella & there is some patellar crepitus. Decreasing myofascial restrictions in which muscle would most likely result in greatest reduction in this Pt's symptoms? A. Vastus lateralis B. Semimembranosus C. Semitendinosus D. Vastus medialis E. Quadriceps

Correct: Answer A: Patello-femoral syndrome is caused by a mal-tracking of the patella. Typically the patella tracks to far laterally, this is usually due to a weak vastus medialis & a tight vastus lateralis. Decreasing myofascial restrictions in the vastus lateralis would have the greatest effect on this Pt's symptoms. Surgical Tx for patello-femoral syndrome consists of a lateral patella release. Answer D: Decreasing myofascial restrictions in the vastus medialis could theoretically worsen this condition. Answer B, C & E: Decreasing myofascial restrictions in other muscles may help, but considering the biomechanics of the problem, it will not have the greatest effect.

A female Pt underwent a R-sided radical masectomy 3 YA for breast cancer. As a result, she has residual swelling in her R upper extremity. The Pt completed a course of chemotherapy & was informed last year that she is cancer free. The Pt wishes to have osteopathic Tx to decrease her edema. The PE reveals pitting edema in the R upper extremity. Which of the following is the most accurate statement regarding lymphatic Tx? A. Proper lymphatic Tx includes, opening the thoracic duct, then releasing the thoracoabdominal diaphragm then applying the posterior axillary fold technique. B. Proper lymphatic Tx includes, a thoraco-abdominal diaphragm release, posterior axillary fold technique & pedal pump C. Lymphatic Tx is absolutely contraindicated in this Pt because of the Hx of cancer D. Lymphatic techniques will be of no use since the anatomy has been altered E. Lymphatic Tx should be postponed until the re-occurence of cancer is R/O.

Correct: Answer A: Proper lymphatic Tx includes, opening the thoracic duct, then releasing the thoracoabdominal diaphragm then applying the posterior axillary fold technique. Answer B: Although variations in Tx vary, this is not the best choice because the Pt should have her thoracic inlet released. The lymph from the R upper extremity will drain into the R lymphatic duct, which transverses the thoracic inlet. Answer C: Lymphatic Tx is not absolutely contraindicated in a Pt w/ a Hx of cancer Answer D: Lymphatic techniques have been shown to help in Pts w/ post surgical edema Answer E: Residual limb edema following radical mastectomy is not uncommon. As long as the edema did not develop or there was no increase in her current edema, the chance of cancer re-occoruance is low.

A Pt has a tenderpoint at the R articular process of C5. What is the typical Tx position for the head & neck in order to treat this tenderpoint using counterstrain? A. Flexed, Sidebent to the left, Rotated to the left B. Extended, Sidebent to the left, Rotated to the left C. Flexed, Sidebent to the right, Rotated to the right D. Extended, Sidebent to the right, Rotated to the right E. Flexed, Sidebent to the left, Rotated to the right

Correct: Answer A: Tenderpoints on articular processes are anterior tenderpoints. Tenderpoints on spinous processes are posterior tenderpoints. Anterior tenderpoints are treated in flexion.

A Pt is found to have a posterior TP on the R at L2. Flexion of the L-Spine increases the asymmetry at this segment, whereas extension restores symmetry. Which of the following is the correct ME (direct) Tx position? A. Flexed, rotated left, Sidebent left B. Flexed rotated right, sidebent right C. Extended rotated right sidebent right D. Extended rotated left sidebent left E. Neutral rotated right, sidebent left

Correct: Answer A: The above Pt has a posterior TP on the R at L2. Symmetry is restored in extension. Therefore the SD is extended, rotated right, sidebent right. A ME Tx is a direct Tx that entails positioning the segment against its restrictive barrier (i.e. reversing all planes of motion). Thus, the correct Tx position in the above Pt is flexed, rotated left & sidebent left.

A posterior fibular head dysfunction can cause direct compression on which of the following peripheral nerves? A. Common fibular B. Obturator C. Sciatic D. Tibial E. Sural

Correct: Answer A: The common fibular nerve (AKA common peroneal nerve) courses posterior to the fibular head. A posterior fibular head dysfunction can exert pressure directly on the common fibular nerve resulting in foot drop. Answer B & C: Fibular head dysfunction will not directly compress the obturator or sciatic nerves. Answer D: The tibial nerve courses through the popliteal fossa, it is not affected by a fibular head dysfunction. Answer E: The sural nerve is made up of fibers from the common peroneal & tibial. It would not be compressed in fibular head dysfunction since these branches arise proximal to the fibular head.

A Pt is Dx'ed w/ a 1st degree ankle sprain. What is the most likely injured ligament? A. Anterior talofibular B. Posterior talofibular C. Calcaneofibular D. Tibiocalcaneal E. Deltoid

Correct: Answer A: The most injured ligament of the foot is the anterior talofibular ligament Answer B & C: The posterior talofibular & calcaneofibular ligaments are less frequently inured & are associated w/ a grade II & III ankle sprains Answer D: The tibiocalcaneal ligament is a portion of the deltoid ligament. It is rarely injured. Answer E: Since the ankle is stable in dorsiflexion & the deltoid ligament very strong, injury to this ligament is rare.

A 21 YO complains of a HA, severe facial pain, greenish purulent nasal discharge & nasal congestion. At which spinal levels would you expect to find the tissue texture changes related to a viscerosomatic reflex? A. T1 - T3 B. T4 - T6 C. T7 - T9 D. T10 - T12 E. T12 - L2

Correct: Answer A: This Pt most likely has sinusitis, & would have viscerosomatic reflex changes at T1 - T3 spinal levels. Answer B: T4 - T6 spinal levels are associated w/ viscerosomatic reflexes from the lungs & esophagus Answer C: T7 - T9 spinal levels are associated w/ the viscersomatic reflexes from the upper GI tract Answer D: T10 - T12 spinal levels are associated w/ the viscerosomatic reflexes from the middle GI tract & the gonads Answer E: T12 - L2 spinal levels are associated w/ the lower GI tract

Which of the following statements concerning Tx of rib dysfunctions is true? A. If a thoracic dysfunction is present it should be treated before rib dysfunctions. B. Exhalation dysfunctions cannot be treated w/ HVLA C. In general, posterior tenderpoints are treated by rotating & sidebending the thorax toward the dysfunction. D. ME techniques are contraindicated in elderly Pt's w/ COPD E. An inhalation dysfunction of ribs 6-9 can be corrected by contracting the serratus anterior muscle.

Correct: Answer A: Thoracic dysfunctions should be treated before rib dysfunctions. Rib dysfunctions may be caused my corresponding thoracic dysfunctions. If a thoracic dysfunction is present, treat the T-Spine before treating the specific rib dysfunction. Answer B: Rib exhalation dysfunctions can be treated using the Kirksville Krunch technique. The technique is the same except that the thenar eminence is placed under the corresponding rib angle. Answer C: In general, posterior rib angle tenderpoints are treated by rotating & sidebending the thorax away from the dysfunction. Answer D: Although ME techniques should not be performed on Pt's w/ low vitality who could be compromised by adding muscular exertion, elderly Pts w/ COPD can typically manage active muscular contractoin w/out exacerbating their COPD. Answer E: An exhalation dysfunction of ribs 6-9 can be corrected w/ ME by contracting the serratus anterior.

A 67 YO male w/ a Hx of CHF presents to the ER in a very anxious state w/ SOB & bilateral lower extremity edema. After appropriate diuresis, the Pt stabilizes. You decide to perform OMT to re-establish homeostasis & enhance lymphatic drainage. Which of the following describes the most appropriate & proper sequencing to Tx? A. Thoracic inlet release, rib raising, redome the diaphragm, pelvic diaphragm release, pedal pump. B. Pedal pump, pelvic diaphragm release, redome the diaphragm, thoracic inlet release, pectoral lift. C. Pedal pump, pelvic diaphragm release, redome the diaphragm, rib raising. D. Rib raising, CV4 technique, pectoral life, pedal pump. E. Redome the diaphragm, pelvic diaphragm release, pectoral lift.

Correct: Answer A: Tx to improve lympatic return should start w/ the removal of all central restrictions (release the diaphragm & rib raising), followed by release of the periphery (lymphatic pumps). The basic lymphatic Tx program includes: 1. Releasing the diaphragms: restrictions w/in the thoracic inlet will obstruct lymph drainage from anywhere in the body. Redoming the diaphragm will produce effective pressure gradients & enhance lymph return. Restrictions w/in the pelvic diaphragm will decrease lower extremity lymphatic drainage. 2. Rib raising to reduce hypersympathetic activity (larger lymph channels receive sympathetic innervation & mobilizing ribs will also enhance respiration) 3. Lymphatic pumps: a pedal pump will promote further lymph return. Answer B & C: As a general rule, diaphragms are first released before applying lymphatic techniques. This ensures the least amount of lymphatic resistance & maximizes the performance of the lymphatic technique (in this case, pedal pump). Answer D: Although rib raising, pectoral lift & pedal pumps will enhance lymphatic drainage, this answer does not list any diaphragm releases & therefore it is not the best answer. Answer E: Not the best answer because it does not address the lower extremity edema.

A 52 YO male w/ atrial fibrilation comes to your office complaining of a new onset R shoulder pain. The pain started 24 hours ago during his golf tournament. The Pt recalls that the pain came about immediately following a golf swing. The pain, which is the greatest at the tip of the R acromion does not radiate. He denies any numbness or tingling in his upper extremity. There is full passive ROM. At rest, w/ his arm by his side, the Pt has little pain. However, the pain becomes moderate to severe when he attempts to hold his arm above his head. X-Rays of his shoulder show no fracture. His medications include Coumadin & hydrochlorothiazide. Given the Pts Hx & physical findings, which of the following Tx's would be innapropriate? A. Ultrasound to the shoulder B. L upper extremity sling C. Ice to the shoulder region D. Fascial release of the shoulder E. Cervical HVLA

Correct: Answer A: Ultrasound, which is a form of deep heat, increases the inflammatory response & will worsen the Pt's edema. Answer B: Ice will help decrease edema following a musculoskeletal injury. It is indicated for the initial management of the injury. Answer C: Although it is not recommended that the Pt wear a sling for more than a week, it can be used to initially provide relative rest depending on the severity of the injury & Pt's activities of daily living. Answer D: A fascial release of the shoulder may be attempted to decrease myofascial restrictions & improve lymphatic return to reduce edema & promote healing. Answer E: HVLA to the C-Spine is not contraindicated in t his Pt. It will not directly affect the glenohumural joint. s

A 34 YO male presents to your office w/ severe low back pain. The pain started yesterday while he was working on his car & is now radiating into his lower extremities. What additional info in this Pt's Hx would most impact the immediate Dx & Tx? A. Absent unilateral superficial abdominal reflex B. Presence of incontinence C. Loss of Achilles reflex D. Down-going plantar response w/ Babinski's testing E. Unilateral loss of cremasteric reflex

Correct: Answer B: A Pt w/ low back pain radiating into both lower extremities may be of neurogenic or musculoskeletal origin. Of all the neurogenic or musculoskeletal possibilities the most important entity to rule out is Cauda Equina Syndrome (CES). CES is an entrapment of terminal nerve roots of the SC. This can be due to central disc herniation. If S2-S4 nerve roots are involved incontinence can result. If this occurs, immediate surgical decompression is indicated. If decompression is delayed, irreversible incontinence may occur. Answer A: Unilateral loss of the superficial abdominal reflex indicates a lower motor neuron lesion was suspected, such as a herniated disc, it may impact his Dx, but his Tx is still likely to be conservative management. Answer C: If the Pt had an absent Achilles reflex, this would indicate a lower motor neuron lesion at S1. If this were present, the Tx would still likely be conservative. Answer D: A downgoing plantar response when trying to elicit a Babinski reflex is a normal adult response. In other words, there is no Babinski reflex. Therefore, there is no suspecgted upper motor neuron lesion. However, the Pt may still have a lower motor neuron lesion present. Answer E: A unilateral loss of the cremasteric reflex suggests a lower motor neuron lesion between L1 & L2. This finding may impact Dx & Tx, however it would not impact the Dx & Tx as much as it would if it were the CES.

A 52 YO male w/ atrial fibrilation comes to your office complaining of a new onset R shoulder pain. The pain started 24 hours ago during his golf tournament. The Pt recalls that the pain came about immediately following a golf swing. The pain, which is the greatest at the tip of the R acromion does not radiate. He denies any numbness or tingling in his upper extremity. There is full passive ROM. At rest, w/ his arm by his side, the Pt has little pain. However, the pain becomes moderate to severe when he attempts to hold his arm above his head. X-Rays of his shoulder show no fracture. His medications include Coumadin & hydrochlorothiazide. The Pt returns in 2 weeks for follow-up. He states that the pain has not improved. AP & lateral X-Rays in your office showed a decreased sub-acromial space w/ a high riding humerus. What is the best radiographic study to determine the extent of this Pt's injury? A. Computed tomography B. MRI C. Bone scan D. Axillary view radiograph E. Pet scan

Correct: Answer B: A decreased sub-acromial space & a high riding humerus raises the likelihood of a complete rotator cuff tear. An MRI is the best imaging study to ID the extent of the injury. Answer A: A CT scan does not show the detail of soft tissue as would an MRI. Answer C & E: A bone scan & PET scan would show general inflammation in the area of the shoulder but would not ID the extend of the soft tissue injury. Answer D: An axillary radiograph is the best view to determine if there is a shoulder dislocation. It will not ID a rotator cuff tear.

A 13 YO male comes to the ED w/ a R wrist & elbow pain following a fall. He states that he fell forward on his outstretched arm. Based on the Hx, what is the most likely Dx at the elbow? A. Adducted ulna B. Posterior radial head C. Abducted ulna D. Cubital tunnel syndrome E. Medial epicondylitis

Correct: Answer B: A posterior radial head may present as a sharp pain at the wrist or elbow & may result from a fall on an outstretched arm. On examination, the posterior radial head will resist anterior glide at the elbow & the forearm will be restricted w/ supination. Answer A, C, D, & E: The other answers listed are possible, however none are associated specifically w/ a fall on an outstretched arm.

A Pt was snowboarding & fell several times. The Pt now complains of L arm pain. The findings of the structural exam are: a decreased carrying angle of the L arm, the olecranonn process is restricted in medial glide, & the wrist & hand are abducted. The most likely Dx is? A. Posterior radial head dysfunction B. Adduction dysfunction of the ulna C. Abduction dysfunction of the ulna D. Anterior radial head dysfunction E. Cervical radiculopathy

Correct: Answer B: An adducted dysfunction of the ulna would have a decreased carrying angle, the hand & wrist would be abducted & the olecrannon process would prefer lateral glide. Answer A: A posterior radial head dysfunction would cause the forearm to prefer pronation, & the radial head would resist anterior glide. Falling on a pronated forearm causes this dysfunction. The carrying angle is not affected in radial head dysfunctions. Answer C: An abducted ulna would have a increases carrying angle; the olecrannon process would be restricted in medial glide, & the wrist & hand would be adducted. Answer D: An anterior radial head dysfunction would cause the forearm to prefer supination & the radial head would resist posterior glide. Falling backward on a supinated forearm can cause this dysfunction. The carrying angle is not affected in radial head dysfunction. Answer E: Cervical radiculopathy is nerve root compression. It can occur w/ trauma, usually to the neck, not to the upper extremity & typically numbness & tingling in the extremity would be present.

A 17 YO female complains of anterior knee pain that gets worse when she runs, jumps or climbs stairs. PE reveals pain is reproducible on squatting & when the knee is flexed. There is point tenderness on the undersurface of the patella & there is some patellar crepitus. The Pt's condition has been most closely associated w/: A. Genu recurvatum B. Increased Q angle C. Decreased Q angle D. Coxa valgus E. Coxa varus

Correct: Answer B: An increased Q angle has a major effect on the tracking of the patella. As the Q angle increases, the Pt's knees become more knock-kneed (genu valgus), causing an abnormal tracking of the patella, predisposing the patella to irregular or accelerated wear. The patella may even sublux laterally w/ these biomechanical forces, especially w/ dysfunction or weakness of the vastus medialis muscle. Answer A: Genu recurvatum is a hyperextension of the knee due to ligament laxity, also called back knee, & has not been associated w/ patello-femoral syndrome. Answer C: A decreased Q angle is not associated w/ patello-femoral syndrome Answer D: Coxa valgus occurs when the angle between the neck & shaft of the femur is greater than 135 degrees. This has no association w/ patello-femoral syndrome. Answer E: Coxa vara has not been associated w/ patello-femoral syndrome

A Pt w/ diverticulitis is likely to have chronic changes related to a Chapman's point in which of the following areas? A. Periumbilically B. Illiotibial band on the L C. Paraspinal muscles at T9 D. SP of T11 E. Paraspinal muscles at L4

Correct: Answer B: Chapman's reflexes are predictable tissue texture abnormalities assumed to be a reflection of visceral dysfunction or pathology. The Chapman's points for the colon is along the femur w/in the iliotibial band. Answer A: The anterior Chapman's point for the adrenals & kidneys are located in the periumbilical region Answer C: Changes in the paraspinals of T9 are too cephalad to represent a viscerosomatic from diverticulosis. L2. Answer D: The posterior Chapman's point for the adrenals is located at SP of T11. Answer E: The SC ends at L2. There are no reported viscerosomatic changes occurring form L3-L5

During evaluation of a Pt's upper back pain, you notice that your R thumb is more posterior upon palpation of the TP of T4. T4 returns to neutral position w/ flexion of the T-Spine, whereas extension of the T-Spine increases the asymmetry. Which of the following is the most correct assessment of T4? A. Sidebent left, Rotated left B. Sidebent right, Rotated right C. Sidebent right, Rotated left D. Sidebent left, Rotated right E. Rotated left, NO Sidebending component

Correct: Answer B: If your R thumb is posterior, this indicates R rotation. Flexion returns T4 to the neutral position indicating a flexed Type II SD. Fryette's pinciples state that in a non-neutral dysfunction sidebending & rotation are to the same side. If a segment is rotated right, then it is also sidebent right. Type II vertebral dysfunctions are typical of single vertebral dysfunctions.

A 34 YO female comes into your office complaining of mild L-sided thoracic pain. The pain started about a week after the Pt began driving her new sports car that has very low riding seats. The pain is non-radiating & worsens w/ inhalation. X-Rays & EKG reveal no abnormalities. Your structural exam reveals that ribs 3 -5 on the L are more caudad & lag behind during inhalation. Tenderpoints are noted on ribs 3 - 5 in the L mid-axillary line. In order to correct the SD, Tx should be directed at which rib? A. Rib 2 B. Rib 3 C. Rib 4 D. Rib 5 E. Rib 6

Correct: Answer B: In a group dysfunction usually one "key" rib is responsible in causing the dysfunction. In exhalation dysfunction the "key" rib is the uppermost rib of the dysfunction, in this case, rib 3. Answer D: If the Pt were to have an inhalation dysfunction, Tx would be directed at rib 5.

A 17 YO high school gymnast comes to your office for a High school physical. Throughout her school career she has been having intermittent back pain. The back pain is localized to the R thoraco-lumbar region & worsens w/ spinal extension. On examination, her R shoulder is lower than her L, & her R iliac crest is higher. Her R leg is 1.2 cm shorter than the left. L2 is flexed, Rotated right & Sidebent right. Her L-Spine appears to be sidebent to the R & L5 is Rotated left. She has full ROM & reverses her scoliotic curve on spinal motion testing. Pelvis shifts easily to the L & resistance is present w/ R pelvic shift. There is a tenderpoint in the center of her L buttock. On standing postural X-ray of the spine & pelvis the scoliotic curve measures 140 & the L femoral head is 12mm caudad compared to the R. A spina bifida occulta is noted on X-rays w/out evidence of spondylolisthesis. Which of the following is the best Tx for this Pt's scoliosis? A. Only OMT would be needed to completely straighten the scoliotic curve B. PT, OMT & Konstancin exercises C. PT, OMT, Konstancin exercises & spinal bracing D. PT, OMT, Konstancin exercises, sprinal bracing & botulism toxin A injections into the paraspinals E. Neurological referral

Correct: Answer B: Mild scoliosis (Cobb angle of 5-150) is typically treated w/ conservative measures. This includes, PT, Konstancin exercises & OMT. Konstancin exercises are a series of specific exercises that has been proven to improve the Pt's w/ scoliotic postural decompensation. Answer A: OMT is not intended to completely straighten scoliotic curves. Answer C: Conservative therapy & spinal bracing are indicated in Pt's w/ moderate scoliotic curves (20-450) Answer D: Botulinum toxin A (BOTOX) injections along w/ conservative management & bracing have been proven to be helpful in some small pilot studies w/ Pt's w/ moderate scoliosis. However, it is not considered the standard of care. Answer E: Neurosurgical referral is indicated for Pt's w/ severe scoliosis.

A Pt complains of epigastric pain that is relieved upon eating food & recurs about 3 hours after he eats. Upper endoscopy is performed & erosions are found in the gastric mucosa. At which spinal level would you expect to find palpatory changes related to a viscerosomatic reflex? A. C7 B. T4 C. T6 D. T10 E. T12

Correct: Answer C: This Pt has peptic ulcer disease. The gastric ulcer would cause viscersomatic changes to occur at the T6 spinal level. Answer A: C7 is not associated w/ viscerosomatic changes related to peptic ulcer disease. Answer B: T4 receives sympathetic innervation from the head & neck, heart, esophagus & lungs. Answer D & E: T10 & T12 receive sympathetic innervation from the middle & lower GI tracts.

A 52 YO male w/ atrial fibrilation comes to your office complaining of a new onset R shoulder pain. The pain started 24 hours ago during his golf tournament. The Pt recalls that the pain came about immediately following a golf swing. The pain, which is the greatest at the tip of the R acromion does not radiate. He denies any numbness or tingling in his upper extremity. There is full passive ROM. At rest, w/ his arm by his side, the Pt has little pain. However, the pain becomes moderate to severe when he attempts to hold his arm above his head. X-Rays of his shoulder show no fracture. His medications include Coumadin & hydrochlorothiazide. In addition to osteopathic Tx, the most appropriate oral medications to treat this Pt's pain is: A. Ibuprofen B. Rofecoxib C. Indomethacin D. Depomedrol E. Hydrocodone

Correct: Answer B: Rofecoxib is a NSAID that achieves its effects by inhibiting COX-2. By selectively inhibiting COX-2 it is less likely to cause a GI bleed. Since the Pt is on coumadin, the Pt will be at greater risk if a GI bleed develops. Another medicine that would be indicated for this Pt is acetaminophen. Answer A & C: Ibuprofen & indomethacin are NSAID's taht would put the Pt at an increased risk for GI bleed. Answer D: Rotator cuff tears are not treated w/ oral steroids due to the numerous adverse effects of this class of medications. The same beneficial effects can be achieved w/ NSAIDs. Answer E: Narcotics are reserved for pain that does not respond to non-narcotic med's.

You are assessing a newborn in the hospital nursery. The child was born yesterday, delivered at 41 weeks by spontaneous vaginal delivery complicated by shoulder dystocia. At birth the child was 10lbs 2oz & measured 21 inches from head to toe. The Apgars were 9 & 10. The mother has been attempting to breast feed, however the child is having difficulty latching on to the breast. On examination, the infant appears to be nervous & irritable. Flexing & abducting the hips while exerting pressure on the greater trochanters demonstrates that the child has a R hip click. What is the most likely cause of the shoulder dystocia? A. Gynecoid pelvic shape B. The child was macrosomic C. The mother did not receive prenatal care D. The obstetrician probably used forceps during delivery E. Premature rupture of membranes

Correct: Answer B: Shoulder dystocia is the inability of the child's shoulder to fit comfortable through the mother's pelvis. Shoulder dystocia is often associated w/ macrosmia. Any neonate weighing over 4000g is considered macrosomic. Answer A: The gynecoid pelvis typically has adequate space along the birth canal when compared to other pelvic types. Therefore it is less likely to cause shoulder dystocia. Answer C & D: Maternal prenatal care & forceps delivery are not associated w/ shoulder dystocia. Answer E: Premature rupture of membranes is not associated w/ shoulder dystocia.

A 13 YO male comes to the ED w/ a R wrist & elbow pain following a fall. He states that he fell forward on his outstretched arm. On examination, there is no evidence of deformity of the forearm or wrist. Upon palpation he has exquisite pain between the extensor longus & abductor pollicis longus tenduons at the radiocarpal joint. There is mild edema at the radiocarpal joint. He has decreased flexion & extension at the wrist as well as decreased forearm supination. There is a tender point located at the musculotendonous insetion of the lateral epicondyle. X-Rays of the wrist & elbow are normal w/out evidence of fracture. The Pt's wrist is splinted & he is told to follow up in 3 weeks. At the next follow up visit, the Pt's pain has not improved. Which of the following would be the appropriate course of action? A. Prescribe a more effective pain med & have the child follow up in a week. B. Order a bone scan C. Immobilize the wrist for 2 weeks w/ a cast D. Reassure the parent & the child that the inflammation & pain are typical of wrist sprains E. None of the above are correct

Correct: Answer B: Since the Pt had pain at the anatomic snuff box & symptoms have not improved in 3 weeks time, the Dx of fracture should be considered. Although X-Rays of the wrist are normal, the Pt may still have a small non-displaced scaphoid fracture. The scaphoid bone is the most common bone fractured in the wrist & often presents w/ pain in the anatomic snuff box. X-Rays may be normal for a week or longer. If a fracture is suspected, the practicioner may obtain a second set of X-Rays, or a bone scan can be ordered. A bone scan will accurately ID a fractured scaphoid. Answer A: Putting the child on a pain med's w/out immobilizing the wrist is ill advised, especially w/ the likelihood of a fracture. Answer C: Appropriate management for a scaphoid wrist fracture is immobilization of the wrist for 3 months. Due to the lack of a rich blood supply, the healing of a fractured scaphoid is characteristically slow. Immobilizing the wrist for 2 weeks will not allow the fracture to fully heal. Answer D: Wrist sprains typically improve w/in a week's time. If the swellling & pain are increased, the Dx of a fracture should be considered.

A 53 YO male comes to the ED complaining of low back pain. He states that the pain he is experiencing now is similar to his back pain that he had when he herniated a disc 2 years ago. He brings in X-rays & an MRI from 2 years ago. X-rays show spondylosis at L5/S1. The MRI demomstrated a L posterior lateral herniated disc at this level. The back pain radiates from the L-Spine into his L foot. Which of the following maneuvers would be expected to decrease this Pt's pain? A. Active lumbar flexion in a standing position B. Passive lumbar extension in the prone position C. Valsalva maneuver D. Straight leg raising E. Ambulation

Correct: Answer B: Symptoms from a herniated dis typically improve / lumbar extension. Therapeutic exercises called the Mckenzie program has been specifically designed to reduce low back pain caused by herniated discs. Most of the McKenzie exercises are extension based maneuvers. The theory behind these extension techniques is that lumbar extension draws the nucleus back into the center of the intervetebral disc Answer A: Active lumbar flexion will push the disc posteriorly, this typically worsens the Pt's pain Answer C: Valsalva maneuvers will typically increase pain associated w/ a herniated disc Answer D: Straight leg raising test will stretch the sciatic nerve, this typically causes the nerve to be pulled over the herniated disc, thus causing pain. Answer E: Ambulation increases axial load & compresses the intervetebral disc resulting in increased pain.

A 34 YO female complains of dyspnea, fatigue, weakness, weight loss & increased appetite. On PE you note a goiter, warm sweaty skin, & a hand tremor. To confirm your Dx you would find an anterior Chapman's point just lateral to the sternum at the interspace of: A. Ribs 1 & 2 B. Ribs 2 & 3 C. Ribs 3 & 4 D. Ribs 4 & 5 E. Ribs 5 & 6

Correct: Answer B: The Pt has hyperthyroidism, & the Chapman's reflex point is associated w/ thyroid dysfunction is found at both interspaces of ribs 2 & 3, just lateral to the sternum. Answer A: Next to the sternum at the interspace of ribs 1 & 2 is the Chapman's point for the tonsils Answer C: Next to the sternum at the interspace of ribs 3 & 4 is the Chapman's point for the upper lung. Answer D: Next to the sternum at the interspace of ribs 4 & 5 is the Chapman's point for the lower lung. Answer E: Next to the sternum at the interspace of ribs 5 & 6 on the L is is the Chapman's point for stomach acidity, & on the R is the Chapman's point for the liver. f

Which of the following biomechanical or structural abnormalities best explain why a Pt w/ COPD has an increased AP diameter of the chest? A. Fibrotic changes w/in the diaphragm due to chronic hypoxia results in a flattened diaphragm & an increased total lung capacity B. Continuous accessory muscle use will pull the ribs superiorly, resulting in an expanded thoracic cage. C. The destruction of elastic fibers & chronic air trapping results in permanent inhaled rib positions D. Chronic deoxygenation of peripheral tissues results in a physiologic expansion of the thoracic cage as a compensatory mechanism. E. Exaggerated sympathetic input from visceral afferents causes broncho-aveolar expansion & results in an increased AP diameter.

Correct: Answer C: Destruction of elastic fibers results from years of smoking (an emphysema type of COPD). Irritation of the lung parenchyma may result in an overproduction of mucus causing obstruction & air trapping (chronic bronchitis & asthma type of COPD). These 2 factors expand the chest cavity & can result in an increased lung capacity, thus permanently positioning the ribs in inhalation. Answer A: Pt's w/ COPD will have impaired oxygenation, however it will not result in a fibrosis & flattening of the diaphragm. If this were true all skeletal muscles in the body would be fibrosed. Answer B: Most accessory muscles will help elevate the ribs into inhalation, however these muscles are not strong enough to overcome the natural recoil of the lungs. Answer D: Chronic deoxygenation will result in a change in minute ventilation, rather than a permanent expansion of the thoracic cage. Answer E: Increased sympathetic tone from visceral afferenets results in bronchodilation & consequently broncho-aveolar expansion. However, this is not the primary reason for an increased AP diameter in a person w/ COPD.

You are assessing a newborn in the hospital nursery. The child was born yesterday, delivered at 41 weeks by spontaneous vaginal delivery complicated by shoulder dystocia. At birth the child was 10lbs 2oz & measured 21 inches from head to toe. The Apgars were 9 & 10. The mother has been attempting to breast feed, however the child is having difficulty latching on to the breast. On examination, the infant appears to be nervous & irritable. Flexing & abducting the hips while exerting pressure on the greater trochanters demonstrates that the child has a R hip click. Which of the following is the most serious neurologic complication that has been associated w/ shoulder dystocia? A. Amblyopia B. Klumpke's palsy C. Erb-Duchenne's palsy D. Bell's Palsy E. Cerebral palsy

Correct: Answer C: Erb-Duchenne's palsy is often due to excessive traction placed on the head & neck while the shoulder is depressed. This can occur when the fetus' shoulders are having difficulty exiting the pelvis. Answer A: Shoulder dystocia is not associated w/ amblyopia. Answer B: Klumpke's palsy is any injury to the lower brachial plexus. It can be due to extreme abduction & extension of the shoulder. It is not associated w/ shoulder dystocia. Answer D: Bell's Palsy results in an injury to the facial nerve & is not associated w/ childbirth. Answer E: Cerebral palsy is primarily a disorder of movement & posture that is due to a non-progressive lesion to an immature brain. It is not associated w/ shoulder dystocia.

In a 41 YO male w/ back pain, you observe that thoracic segments T8 - T12 have a convexity to the R. Which of the following is the best statement regarding this groups SD? A. T9's R TP is more anterior B. T12 is limited in R Rotation compared to L1 C. T10's L TP is more caudad compared to its R D. Ribs 7 - 12 are limited in exhalation on the L E. T9 is Rotated right, Sidebent right

Correct: Answer C: If T8 - T12 have a convexity to the R, there is a group dysfunction that is Sidebent left & Rotated right. Remember group dysfunctions occur in the neutral plane & in such a case they Sidebent & Rotate to opposite sides. If T10 is Sidebent left, then its left TP will be more caudad than its R TP. Answer A: If T9 is Rotated to the right, then its R TP would be more posterior Answer B: If T12 is Rotated right, it will be limited in L Rotation. Answer D: A thoracic dysfunction does not always preclude a rib dysfunction. There is not sufficient info given to determine if a rib dysfunction is present. Answer E: The question describing a neutral dysfunction that is Rotated right, Sidebent left. Therefore T9 cannot be Rotated right & Sidebent right

Plexopathy involving the medial cord of the brachial plexus will have the greatest effect on which of the following movements? A. Elbow flexion B. Elbow extension C. Finger abduction D. Shoulder abduction E. Shoulder external rotation

Correct: Answer C: Injury to the medial cord of the brachial plexus is likely to affect C8, T1 & muscles innervated by the median & ulnar nerves. The interossi muscles of the hand are responsible for finger abduction. The ulnar nerve innervates the interossi. Answer A: The musculotaneous nerve is responsible for elbow flexion. This nerve originates from the lateral cord. Answer B: The radial nerve is responsible for elbow extension. This nerve originates from the posterior cord. Answer D: The axillary nerve is primarily responsible for shoulder abduction. This nerve originates from the posterior cord. Answer E: The teres major (axillary nerve) & infraspinatus (suprascapular nerve) are responsible for shoulder external rotation. These nerves originate from the posterior & lateral cords respectively.

A 34 YO female comes into your office complaining of mild L-sided thoracic pain. The pain started about a week after the Pt began driving her new sports car that has very low riding seats. The pain is non-radiating & worsens w/ inhalation. X-Rays & EKG reveal no abnormalities. Your structural exam reveals that ribs 3 -5 on the L are more caudad & lag behind during inhalation. Tenderpoints are noted on ribs 3 - 5 in the L mid-axillary line. If a ME technique is used to treat this dysfunction, which muscle should be recruited? A. Posterior scalene B. Teres major C. Pectoralis minor D. Pectoralis major E. Serratus anterior

Correct: Answer C: Ribs 3-5 are held down in exhalation. The pectoralis minor muscle originates from the coracoid process & inserts on ribs 3-5. Activation of this ME would pull the shaft of the ribs superior, correcting the dysfunction. Answer A: The posterior scalene is used to treat a SD of rib 2 Answer B & D: The teres major & pectoralis major are not muscles used to treat rib dysfunctions Answer E: The serratus anterior is used to treat a SD of ribs 6-9

W/ deep inspiration, ribs 6 - 10 increase the chest wall diameter: A. Anteriorly along an oblique plane B. Anteriorly around a transverse axis C. Laterally around an anterior - posterior axis D. Laterally around a transverse axis E. Laterally around a vertical axis

Correct: Answer C: Ribs 6 - 10 primarily move in a bucket - handle motion. Movement is around an AP axis, therefore, inhalation will increase the transverse diameter of the chest wall. Answer A: Physiologic rib motion does not occur along an oblique plane. Answer B: Pump-handle ribs will increase the chest wall anteriorly around a transverse axis. Answer D: Rib movement around a transverse axis could not expand the ribcage laterally. Answer E: Ribs do not move around a vertical axis.

Which of the following is true regarding the anatomy & biomechanics of the thoracic region? A. When a Pt is standing w/ their arms at their side, the medial border of the spine of the scapula is immediately lateral to the SP of T1 B. The SP of the thoracic region point increasingly downward, such that T1's SP is approximately at the level of the TP of T2. C. Restriction w/in Sibson's fascia is likely to limit lymphatic drainage form the L lower extremity. D. The main motion of the thoracic cage is about a transverse axis E. At quiet breathing the diaphragm bears 80-90% of the workload

Correct: Answer C: Sibson's fascia forms the functional cervico-thoracic diaphragm (thoracic inlet) & is especially involved in the mechanics of fluid homeostasis in the entire body. The thoracic duct, which drains the L upper extremity & bilateral lower extremities, passes through the thoracic inlet. Therefore restriction w/in the thoracic inlet is likely to limit lymphatic drainage of the L lower extremity. Answer A: When a Pt is standing w/ their arms at their side, the medial spine of the scapula is immediately lateral to the SP of T3, not T1. Answer B: Although the SP point increasingly downward, the rule of 3's dictates that the SP of T1 is located equal to the TP of T1. Answer D: The main motion of the T-Spine is rotation. Rotation moves about a vertical axis. Answer E: At quite breathing the diaphragm bears 100% of the workload. During exercise the diaphragm bears 60% of the workload; and the secondary muscles of respiration are responsible for 40% during exercise.

A 53 YO male comes to the ED complaining of low back pain. He states that the pain he is experiencing now is similar to his back pain that he had when he herniated a disc 2 years ago. He brings in X-rays & an MRI from 2 years ago. X-rays show spondylosis at L5/S1. The MRI demomstrated a L posterior lateral herniated disc at this level. The back pain radiates from the L-Spine into his L foot. The X-Ray finding in the above Pt is most consistent w/? A. Anterior slippage of the L5 vertebrae on the sacrum B. A defect in the pars interarticularis C. Degenerative changes & osteophytic lipping D. Ankylosing spondylitis

Correct: Answer C: Spondylolysis is a radiological term for degenerative changes in the intervetebral discs & anklyosing of the adjacent vertebral bodies. Answer A: Anterior slippage of the L5 vertebrae on the sacrum is spondylolisthesis, not spondylosis. Answer B: A defect in the pars interarticularis is spondylolysis, not spondylosis. Answer D: Spondylosis is not associated w/ compression fractures. Answer E: Ankylosing spondylitis is a systemic disease that results in akylosing of several vertebrae starting at the lumbosacral spine & proceeding cephalad. Although spondylosis can include ankylosing of adjacent vertebral bodies, a 53 YO male w/ ankylosing spondylitis would have several joints involved, not just at L5/S1

You are assessing a newborn in the hospital nursery. The child was born yesterday, delivered at 41 weeks by spontaneous vaginal delivery complicated by shoulder dystocia. At birth the child was 10lbs 2oz & measured 21 inches from head to toe. The Apgars were 9 & 10. The mother has been attempting to breast feed, however the child is having difficulty latching on to the breast. On examination, the infant appears to be nervous & irritable. Flexing & abducting the hips while exerting pressure on the greater trochanters demonstrates that the child has a R hip click. Which osteopathic Tx is indicated to treat this newborns suckling problem? A. Rib raising B. CV4 technique C. Condylar decompression D. Sphenopalatine ganglion release E. Temporal rocking

Correct: Answer C: Suckling dysfunction can be associated w/ condylar compression. The Tx of choice is condylar decompression.

A 33 YO female comes to your office w/ a 3 day Hx of severe HA. The pain originates from the base of the occiput & radiates into the R temporal region. The Pt reports that she started experiencing nausea, vomiting & vertigo today. Pain intensifies in brightly lit rooms. She smokes 1 pack/day. Her allergies include ragweed. Osteopathic structural examination reveals tenderness in the R trapezius & R temporalis muscles. Palpation of these tender areas does not refer pain. There is a tenderpoint at the articular pillar of C2 on the R. Which of the following medications is most appropriate for initial Tx of this Pt's HA's? A. Butalbital B. Valproic acid C. Sumatriptan D. Propranolol E. Atenolol

Correct: Answer C: Sumatripan (Imatrex) can be used to treat acute migraines. Answer A: Tension HA's are typically treated w/ butalbital in combination w/ acetominophen. Butalbital is a barbituate. Answer B, D & E: Valproic acid (depakote), atenolol (tenormin) & propranolol (Inderal) can be used for migraine prophylaxis.

A Pt complains of mid-thoracic back pain for 3 days. While evaluating T5 you find that your R thumb is more anterior than your L, & asymmetry is not restored after flexion or extension. What is the Dx for this SD? A. T5 Flexed, Rotated right, Sidebent right B. T5 Extended, Rotated right, Sidebent left C. T5 Neutral, Rotated left, Sidebent right D. T5 Neutral, Rotated right, Sidebent left E. T5 Extended Rotated right, Sidebent right

Correct: Answer C: T5 neutral, Rotated left, Sidebent right is the correct Dx. When your thumb is more anterior, this implies that your L thumb is more posteior. This is a L rotation. Testing in flexion & extension did not restore symmetry to T5, so it is neutral in the sagittal plane. By Fryette's 1st law, when the segment is in the neutral plane, sidebending occurs to the opposite side of the rotation. Answer A: In a segment that is Flexed, Rotated right, Sidebent right, your L thumb would be posterior & symmetry would be restored in flexion Answer B: In a extended, Rotated left, Sidebent left, symmetry would be restored in extension. Answer D: In a neutral, Rotated right, Sidebent left, your thumb would be posterior Answer E: In a extended, Rotated right, Sidebent right symmetry would be restored in extension.

A 47 YO female presents to your office w/ low back pain. Upon inspection of the L-Spine, you observe a L lateral convexity. Palpation reveals fullness in teh L paraspinal musculature & L2 - L5 have TP's posterior on the L. Given the above info the SD is most consistent w/: A. Type II mechanics w/ the L-Spine Sidebent left, Rotated left B. Type II mechanics w/ the L-Spine Sidebent right, Rotated right C. Type I mechanics w/ the L-Spine Sidebent right, Rotated left D. Type I mechanics w/ the L-Spine Sidebent left, Rotated right. E. There is not enough info to make a correct Dx

Correct: Answer C: The Pt has a L lateral convexity. In other words, the L-Spine is Sidebent to the R. You also note on palpation that there is a L paraspinal fullness & L2 - L5 (i.e. L2 - L5 is Rotated to the L) have TP's posterior on the L. When the spine sidebends & rotates in opposite directions, we call this type I mechanics. Type I mechanics occur when the spine is in the neutral position (no flexion or extension) & are typical of group dysfunctions. Type II mechanics occur when the spine is either flexed or extended, in this case Sidebending & Rotation would be in the same direction. Type II mechanics are typical of single vertebral dysfunctions.

A 17 YO high school gymnast comes to your office for a High school physical. Throughout her school career she has been having intermittent back pain. The back pain is localized to the R thoraco-lumbar region & worsens w/ spinal extension. On examination, her R shoulder is lower than her L, & her R iliac crest is higher. Her R leg is 1.2 cm shorter than the left. L2 is flexed, Rotated right & Sidebent right. Her L-Spine appears to be sidebent to the R & L5 is Rotated left. She has full ROM & reverses her scoliotic curve on spinal motion testing. Pelvis shifts easily to the L & resistance is present w/ R pelvic shift. There is a tenderpoint in the center of her L buttock. On standing postural X-ray of the spine & pelvis the scoliotic curve measures 140 & the L femoral head is 12mm caudad compared to the R. A spina bifida occulta is noted on X-rays w/out evidence of spondylolisthesis. What is the most likely cause of this Pt's scoliosis? A. Idiopathic scoliosis B. Central spasticity C. Psoas syndrome D. Spina bifida E. Muscle weakness

Correct: Answer C: The Pt has some symptoms of R psoas syndrome (high lumbar dysfunction, pelvic shift to the left & left piriformis tenderpoint, which can be a cause of scoliosis. Answer A: Although idiopathic scoliosis accounts for most cases (80%), in this case there is a musculoskeletal cause. Answer B: Central spasticity indicates an upper motor neuron origin, thus unless there was a Hx of cerebral or SC injury this would not be the most correct answer. Answer D: Spina bifida occulta has not been associated w/ scoliosis Answer E: This PT has not Hx of muscle weakness

As a sports medicine physician you are consulted on a 23 YO female w/ patello-femoral tracking syndrome. In addition to medication & osteopathic Tx you decide to facilitate the Pt's recovery by recommending exercises to strengthen the vastus medialis. You inform her that while in the supine position w/ the knee fully extended, she should tighten her quadriceps, hold for 5 seconds then relax. Which of the following statements most accurately describes this type of exercise? A. Eccentric contraction in which there is no increase in Vastus Medialis tension B. Concentric contraction in which there is an increase in Vastus Medialis tension. C. Isometric contraction in which there is an increase in Vastus Medialis tension. D. Eccentric contraction in which there is a shortening of the muscle. E. Isotonic contraction in which there is an increase in Vastus Medialis tension.

Correct: Answer C: The Pt is performing a type of isometric contraction. The definition of isometric contraction is a muscle contraction that results in an increase in tension w/out a change in length. Answer A, B & D: Concentric & eccentric contractions are contractions in which there is an approximation (concentric) or lenghening (eccentric) of muscular origin & insertion. Tension is not related to these type of contractions. Answer E: An isotonic contraction is one in which there is an approximation of the muscle's origin & insertion w/out an increase in muscle tension. The difference between concentric & isotonic contractions is that concentric contractions can have variable tensions.

A 34 YO male is complaining of severe R-sided low back pain radiating into the foot. The pain started yesterday & is associated w/ calf-cramping. He describes a shooting pain into his R-foot that is made worse w/ forward flexion of the L-Spine. On examination, the pelvis is shifted to the L. There is an absence of lumbar lordosis. The L-Spine is Sidebent to the R. The sacrum has Rotated right around a T oblique axis. There is a positive straight leg raising test. Which of the following statements is true? A. This Pt's injury would have been prevented if he had received frequent OMT B. The primary cause of this Pt's pain is due to a shortened psoas on the R side C. This Pt is likely to have asymmetry w/ hip drop testing D. This PT will eventually need surgery due to the natural progression of symptoms associated w/ his Dx. E. Due to the severity of the pain, initial analgesia should consist of oral opiods.

Correct: Answer C: The above Pt most likely has an acute herniated disc & subsequently has a lumbar sidebending dysfunction. Asymmetry will present w/ hip drop testing w/ a lumbar sidebending dysfunction. Answer A: Currently there is no literature suggesting frequent OMT prevents herniated discs. Answer B: Although the pelvis can be shifted to the L in a R psoas syndrome, a pelvic shift is also a common finding in a person w/ a herniated disc. In addition, psoas syndrome is not associated w/ a positive straight leg test & radiation of pain into the foot. Answer D: Most herniated discs do not require surgery (approximately 95% can be treated conservatively). Answer E: According to the WHO, non-malignant pain should be initially treated w/ non-narcotic analgesia. If this fails, narcotics can then be initiated.

A Pt comes to your office after "his back went out" earlier that morning. Standing postural x-rays determine that there is significant sacral base unleveling. Which ligament is thought to be the 1st ligament to become painful in lumbosacral decompensation? A. Sacrotuberous B. Anterior sacroiliac C. Iliolumbar D. Posterior sacroiliac E. Sacrospinous

Correct: Answer C: The iliolumbar ligament is often the 1st ligament to become painful in lumbosacral decompensation. Answer A, B, D, & E: Sacrospinous, sacrotuberous & sacroiliac ligaments may become painful in lumbosacral decompensation, however they are not the first.

A Pt w/ low back pain received OMT. The following was done: -HVLA to C3, C5, T4, L1, L5 & R innominate -ME to the sacrum -Counterstrain to the piriformis muscle Based on the above info, what is the maximum number of treated areas that should be filled out on the Osteopathic SOAP note form? A. 1-2 areas B. 3-4 areas C. 5-6 areas D. 7-8 areas E. 9-10 areas

Correct: Answer C: The osteopathic SOAP note form is a standardized form that has been adopted by osteopathic schools. Senior med students & interns should be familiar w/ this form. On the form there are 10 regions of the body for examination & Tx. 1. Cranium 2. Cervical 3. Thoracic 4. Ribs 5. Abdomen 6. Lumbar 7. Sacrum 8. Pelvis 9. Upper extremity 10. Lower extremity. There were a total of 6 regions treated. Tx of more than one dysfunction in a region only counts as 1 region. The piriformis muscle has its origin on the sacrum, & its insertion on the femur. It is considered primarily an external rotator of the lower extremity. Tx of this muscle involves using the lower extremity & can therefore be considered teh 6th region.

A Pt in severe unrelenting pain from a recently herniated disc in the L-Spine seeks Tx for his condition. The Pt has a Hx of peptic ulcer disease & DVT. The Pt is currently taking coumadin daily. What would be the most appropriate management of this Pt? A. NSAID, bed rest for 5 days, indirect techniques B. NSAID, bed rest for 2 days, counterstrain C. Opiod analgesic, bed rest for 2 days, counterstrain D. Acetaminophen, bed rest for 5 days HVLA E. Opiod analgesic, bed rest for 5 days, ME

Correct: Answer C: This Pt is in severe pain. Usually, first line med's are NSAIDs. However in this case, they are contraindicated because the Pt is at high risk fro a GI bleed (a Hx of peptic ulcer disease complicated by the fact that he is on coumadin). Therefore given his level of pain, an opiod is indicated. Deyo & colleagues demonstrated that bed rest for 2 days helped Pt's return to work quicker & did not worsen their back pain. In addition, prolonged bed rest in the above Pt may predispose him to forming another DVT. A Pt w/ this level of pain is well suited for gentle techniques such as indirect techniques. Answer A & B: NSAIDs are contraindicated in this Pt due to the high risk of GI bleed Answer D: 5 days of bed rest would not benefit the PT any more than 2 days & HVLA may injure the Pt Answer E: Deyo & colleagues demonstrated that bed rest for 2 days did not worsen Pt's back pain & helped Pt's return to work quicker. Therefore, this is not the best answer.

Match the description with the best term: A 70 YO male w/ rigidity, resting tremor, bradykinesia has a slow walking speed & his feet do not clear the ground. A. High steppage gait B. Ataxic gait C. Shuffling (festinating) gait D. Waddling gait E. Hemiplegic gait F. Elevated pelvis gait G. Antalgic gait H. Gluteus medius gait I. Scissor gait

Correct: Answer C: This is typically described as short, flat-footed shuffling steps in which the foot does not clear the ground. In Parkinsonism, rigidity, tremor, paucity of movement, shuffling w/ haste & difficulty in starting, stopping, or turning are also seen.

A 19 YO male w/ no Hx of past medical illness comes to your office w/ neck pain. Active neck ROM reveals painful R rotation to 45 degrees, & painless L rotation to 90 degrees. Which of the following statements is true regarding this Pt's ROM? A. Since the Pt has limited R rotation he likely has limited L sidebending B. Since the Pt has limited R rotation he likely has limited R sidebending C. This Pt is likely to have decreased passive ROM to the R D. The Pt is likely to have full passive ROM w/ R rotation to the physiologic barrier E. This Pt demonstrated that he has full L rotation to the anatomic barrier.

Correct: Answer C: When you ask a Pt to actively move a joint to it's end points you are asking the Pt to demonstrate active ROM. Passive ROM is when a physician ranges a joint. Any person can actively move his/her joint to its physiologic barrier (assuming no SD). Any other person can move someone else's joint to its anatomic barrier (assuming no SD). When there is SD present, that person or physician can only move the joint to the restrictive barrier. Thus, all motion is lost beyond the restrictive barrier. Answer A & B: The question does not state which segment(s) is/are restricting motion. Therefore, it cannot be determined if the dysfunctional segment follows type I or type II mechanics. Answer D: All motion, active or passive, is lost beyond the restrictive barrier. Answer E: The Pt demonstrated full L rotation to the physiologic barrier, not the anatomic barrier.

During evaluation of a Pt's upper back pain, you notice that your R thumb is more posterior upon palpation of the TP of T4. T4 returns to neutral position w/ flexion of the T-Spine, whereas extension of the T-Spine increases the asymmetry. Which of the following organs would be LEAST affected by an autonomic imbalance at T4? A. Heart B. Esophagus C. Stomach D. Lungs E. Tongue

Correct: Answer C: the stomach generally receives autonomic innervation from T5-T9. Although there is some overlap, out of the other answers T4 is least likely to affect the stomach. Answer A: The heart receives autonomic innervation from T1-T5 Answer B: The esophagus receives autonomic innervation from T2-T8 Answer D: The respiratory system receives autonomic innervation from T2-T7 Answer E: The tongue is a head structure & will receive autonomic innervation from T1-T4

Match the description with the best term: A Pt w/ a Hx of muscular dystrophy has weakness of the pelvic girdle muscles, increased lordosis, pot-bellied posture & appears to roll from side to side when he walks. A. High steppage gait B. Ataxic gait C. Shuffling (festinating) gait D. Waddling gait E. Hemiplegic gait F. Elevated pelvis gait G. Antalgic gait H. Gluteus medius gait I. Scissor gait

Correct: Answer D: A waddling gait can be described as rolling from side to side. The pelvic rotation & pelvic tilt are increased. This has been described as a penguin walk. Muscular dystrophy w/ weakness of hips, exaggerated lordosis, & pot bellied posture can produce this gait.

A Pt complains of CP, especially w hen she exhales. The pain is localized to ribs 8-10 on the L. Upon further assessment of ribs 8 - 10 the lower edge of the rib shaft is more prominent & more elevated laterally than anteriorly. There was a failure of these ribs to move during exhalation. What is the most likely Dx? A. Ribs 8-10 exhalation dysfunction, pump handle predominant B. Ribs 8-10 exhalation dyfsfunction bucket handle predominant C. Ribs 8 - 10 inhalation dysfunction, pump handle predominant D. Ribs 8 - 10 inhalation dysfunction, bucket handle predominant E. Ribs 8 - 10 inhalation dysfunction, caliper motion predominant.

Correct: Answer D: Bucket handle ribs 8 -10 that have an inhalation dysfunction do not move caudally in exhalation & may cause pain during the exhalation phase of respiration. Upon static assessment, the lower edge of the rib shaft is more prominent & elevated laterally. Answer A & B: These are incorrect because the ribs are not in exhalation dysfunction Answer C: If the ribs had a pump handle dysfunction, they would be elevated anteriorly. Answer E: Laterally elevated ribs are more closely associated to bucket handle dysfunctions, not caliper dysfunctions.

You are assessing a newborn in the hospital nursery. The child was born yesterday, delivered at 41 weeks by spontaneous vaginal delivery complicated by shoulder dystocia. At birth the child was 10lbs 2oz & measured 21 inches from head to toe. The Apgars were 9 & 10. The mother has been attempting to breast feed, however the child is having difficulty latching on to the breast. On examination, the infant appears to be nervous & irritable. Flexing & abducting the hips while exerting pressure on the greater trochanters demonstrates that the child has a R hip click. Which of the following SD's has been linked w/ suckling difficulties in the newborn? A. Upper thoracic dysfunction B. Hyperparasympathetic tone C. Maxilae dysfunction D. Cranial nerve dysfunction E. TMJ dysfunction

Correct: Answer D: Suckling difficulties have been associated w/ condylar compression and/or restrictions at the jugular foramen. Condylar compression causes CNXII dysfunction. Other possible causes of suckling dysfunctions have been associated w/ restrictions at the jugular foramen causing CNIX & X dysfunction.

Head & neck lymphatic congestion & drying of the nasopharyngeal mucosa is most likely associated w/ increased autonomic activity originating from the pre-ganglionic nerve fibers of which structure? A. Vagus nerve B. Inferior cervical ganglia C. Superior cervial ganglia D. Intermediolateral cells of the SC at T1 - T4 E. Intermediolateral cells of the SC at T5 - T7

Correct: Answer D: Hypersympathetic activity to the head & neck structures will lead to vasoconstriction producing lymphatic congestion. It will also inhibit secretion from the nasopharyngeal mucous membranes that produces dryness. Sympathetic nerve fibers arise from the intermediolateral cells of the SC. Although it has nothing to do w/ this question, hypersympathetic activity to the respiratory epithelium (lungs) will lead to increased secretion via increased goblet cell production. Answer A: The parasympathetic fibers (from the vagus nerve) will not lead to nasal & pharyngeal dryness or lymphatic congestion. Answer B & C: Postganglionic fibers that supply the head & neck structures stem from the cervical chain ganglia, however pre-ganglionic fibers originate from the interomediolateral cell column in the SC. Answer E: The autonomic nervous system that supplies the head & neck do not originate from T5 - T7.

In a Pt w/ a Hx of tension HA's, structural exam reveals the following: C5 resists lateral translation to the L. There is a right-sided fullness at C2 & the R articular pillar of C2 resists anterior glide. Given the above info, what else is most likely true regarding PE? A. C2 is flexed B. C2 is Rotated left C. The AA joint is Rotated right D. C5 is Rotated left E. The atlas is Rotated left

Correct: Answer D: If C5 resists lateral translation to the L, this indicates that C5 resists Right Sidebending. If it resists R sidebending, ti is Sidebent left. If it is Sidebent left, it must be Rotated left. C5 follows the rules of Type II mechanics. Answer A: Since the question did not test C2 in flexion & extension, we do not know whether C2 is flexed, extended or neutral. Answer B: A common way to test for Rotation in the C-Spine is to push anterior on the articular pillars of the C-Spine at each segment. If there is resistance on one side compared to the other then the segment is Rotated toward that side. In order to test Rotation at C2 the practitioner would push anterior on each articular pillar. The above Pt has paraspinal fullness on the R & the R articular pillar resists anterior glide, this indicates that C2 is Rotated to the R. Answer C & E: The above question does not describe the direction of the AA joint (C1 on C2)

In a Pt w/ low back pain, L5 is flexed & sidebent right. Which the following is true? A. You would expect L5 to rotate easily to the left, based on the laws of type II spina mechanics B. You would expect L5 to sidebend easily to the R, based on the laws of type I spinal mechanics C. You would expect L5 to resist R rotation in the flexed position. D. You would expect the R TP of L5 to become more posterior as the spine moves from flexion to extension. E. You would expect the sacrum to be rotated forward on a L oblique axis if a torsion was present

Correct: Answer D: In the above Pt, L5 is flexed sidebent right & rotated right. Upon palpation of L5's TP, in the neutral position the R TP should be posterior. W/ flexion the asymmetry should resolve. W/ extension however, the asymmetry will re-appear. Answer A: You would expect L5 to rotate easily to the R, based on the laws of type II spinal mechanics. Answer B: Since L5 is sidebent & rotated to the R, it will follow the laws governed by Type II spinal mechanics. Answer C: Since L5 is sidebent & rotated to the R, there should be an ease of R rotation w/ the segment in the flexed position. Answer E: If a sacral torsion was present, the sacrum would be rotated L on a R oblique axis (backward sacral torsion).

A 33 YO female comes to your office w/ a 3 day Hx of severe HA. The pain originates from the base of the occiput & radiates into the R temporal region. The Pt reports that she started experiencing nausea, vomiting & vertigo today. Pain intensifies in brightly lit rooms. She smokes 1 pack/day. Her allergies include ragweed. Osteopathic structural examination reveals tenderness in the R trapezius & R temporalis muscles. Palpation of these tender areas does not refer pain. There is a tenderpoint at the articular pillar of C2 on the R. What is the rational for using rib raising on this Pt? A. It will increase sympathetic tone thus promoting cranial vasoconstriction B. It will decrease sympathetic tone thus promoting cranial vasoconstriction. C. It will increase sympathetic tone thus limiting cranial vasoconstriction D. It will be of no benefit to this Pt

Correct: Answer D: Increased sympathetic tone to innervated BV's of the cranium will cause vasoconstriction. The purpose of rib raising is to reduct hypersympathetic tone thus limiting vasoconstriction. Other SD's, such as cranial dysfunctions & axial dysfunctions may produce fascial strains that are transmitted to the head & may play a role in the origin of a migraine. Answer E: Since classic migraines are thought to be caused by a vasoconstriction of innervated BV's & a vasodilation of non-innervated BV's, techniques aimed at reducing sympathetic tone will decrease initial vasoconstriction.

A Pt w/ renal failure for 8 months would be expected to have: A. Acute tissue texture changes at T7 B. Chronic tissue texture changes at T9 C. Acute tissue texture changes at T11 D. Chronic tissue texture changes at T11 E. Acute tissue texture changes at T9

Correct: Answer D: It is generally thought that chronic tissue texture changes occur after approximately 6 months of dysfunction Acute renal failure is defined as reversible, & chronic renal failure is defined as irreversible renal failure. This will cause chronic tissue changes at the T11 segment. Answer A, C & E: these are incorrect because it is a chronic condition, not acute, and they are at the wrong spinal level. Answer B: Chronic changes at T9 would be associated w/ chronic disease of the upper/middle GI tract.

Which of the following is true regarding the sympathetic nervous innervation to the kidneys & ureters? A. Sympathetic innervation arises from cell bodies in the SC at the level of T8 - T10 B. Increased sympathetic tone causes vasoconstriction of afferent arterioles thus increasing the glomerular filtration rate. C. Stimulating sympathetics will restore normal peristaltic waves in the ureters D. Preganglionic fibers for the kidney & upper ureters synapse in the superior mesenteric ganglion. E. Sympathetic stimulation contracts the detrusor & relaxes the trigone resulting in micturition.

Correct: Answer D: Preganglionic fibers for the kidney & upper ureters synapse in the superior messenteric ganglion, while fibers for the lower ureter synapse on the inferior mesenteric ganglion. Answer A: Sympathetic innervation arises from the cell bodies in the SC at the level of T10 - L1 Answer B: Sympathetic stimulation causes vasoconstriction of afferent arterioles thus decreasing the glomerular filtration rate. Answer C: The parasympathetics will maintain normal peristaltic waves in the ureters, whereas sympathetic stimulation could cause ureterospasm. Answer E: Sympathetic stimulation contracts the trigone (spincter), relaxes the detrusor (bladder wall) resulting in urinary retention.

A Pt comes to the ED complaining of wrist & elbow pain. Yesterday while at work, he fell on his outstretched arm hyperextending his wrist. There is no Hx of direct trauma to the elbow. The wrist is warm, slightly edematous & there is significantly decreased ROM of the wrist & forearm. Palpatory exam reveals severe pain on the volar aspect of the wrist at the ulna-carpal junction. Pulses are normal & sensation is intact. X-Rays of the wrist show some soft-tissue swelling surrounding the wrist but there is no evidence of fracture or carpal dislocation. X-Rays of the elbow are negative. Which of the following is the best statement regarding initial management of this Pt's injury? A. A bone scan should be ordered to R/O a scaphoid fracture. B. Since this Pt is likely to have a fracture of the scaphoid bone, he should be treated empirically by casting his wrist & forearm for 4 - 6 weeks. C. Due to the likelihood of a non-displaced fracture of the scaphoid, this Pt should be empirically treated w/ closed reduction & external fixation w/ percutaneous pinning. D. Due to the acute nature of this illness, the Pt's wrist should be immobilized w/ a soft splint & a thoracic inlet release & pectoral lift can be done to reduce anema. E. Pain at the volar aspect of the wrist most likely represents an ulnar SD. This can be easily treated w/ ME & HVLA to restore ulnar carpal glide.

Correct: Answer D: The Pt is likely to have suffered a severe wrist sprain. Initial management should include edema reduction along w/ imobilization until pain free ROM can be restored. Answer A, B & C: Although 10-20% of Pts w/ a scaphoid fracture will have negative X-Rays, pain is usually present in the anatomic snuffbox, since the scaphoid is located on the radial side of the wrist. The above Pt has pain located on the volar aspect of the ulnar-carpal joint. This is not typical of a scaphoid fracture. Answer E: There is no evidence in the osteopathic literature that specifies pain at the volar aspect of the wrist most likely represents an ulnar SD. In addition, ME & HVLA are contraindicated in acute strains & sprains.

A 72 YO male comes to your office w/ chronic low back pain. He has a 20+ year Hx of diabetes & was recently Dx w/ prostate cancer. His back pain is a dull ache that radiates into the R buttock & thigh. The pain is made worse w/ standing or walking. Pain is relieved w/ sitting. On PE, pulses are strong in the lower extremities & capillary relief is adequate. ROM of his L-Spine is decreased. Sensation is decreased in both feet in a stocking like distribution. Muscle testing is 5/5 bilaterally in lower extremities. Reflex testing is 2+ at the patella bilaterally, & absent at the achilles bilaterally. Considering the Pt's symptoms, what is the most likely finding this Pt will have on radiological studies? A. A herniated nucleus pulposus on MRI B. Lytic lesions involving the lumbar vertebrae C. Spondylolisthesis of L5 on S1 D. Spondylosis of the L-Spine E. Compression fracture of the lumbar vertebrae

Correct: Answer D: The Pt's aggravating factors & alleviating factors along w/ the physical findings should indicate that the Pt most likely has lumbar spinal stenosis. Lumbar stenosis is narrowing of the spinal canal and/or intervetebral foramina. As the Pt extends his L-Spine the intervetebral foramen physiologically becomes more narrowed. If a Pt has narrowing of this area neural impingement can occur. Pt's symptoms will usually improve w/ flexion of the L-Spine. Radiographically, degenerative changes are present in Pt's w/ spinal stenosis. Spondylosis is a radiographic term for general degenerative changes. It is virtually a universal finding in elderly Pt's (even w/out spinal stenosis). Answer A: A herniated disc is possible, however it is less likely because the pain is made better w/ lumbar flexion. The absent achillies reflexes is likely due to diabetic peripheral neuropathy. Answer B: Although we do not know the extent of this Pt's prostate cancer, the likelihood of him having vertebral metastasis (usually seen as lytic lesions) is far less than the likelihood of spondylosis. Answer C: Spondylolisthesis is a much less frequent finding than spondylosis in the elderly Pt. Answer E: Compression fractures of the L-Spine are far less common than spondylosis. In addition, spondylitic changes are usually seen w/ & often occur before compression fractures.

An 85 YO male w/ a Hx of severe cervical degenerative joint disease comes to your office. He states that his neck pain started several years ago & has gotten progressively worse. The pain radiates into the upper extremity & fingertips. Pain is worsened w/ C-Spine extension. What are some of the findings you would expect to see when examining the C-Spine? A. Boggy, edematous paraspinal musculature w/ full ROM of the C-Spine. B. Warm & moist skin texture w/ some erythema. Sharp, painful paraspinal musculature, & decreased ROM of the C-Spine. C. Hypertonic paraspinals w/ severe, sharp pain w/ palpation. And, a compensated spinal curve in the T-Spine D. Dry skin, & a moderate amount of paraspinal tenderness w/ ropy & fibrotic cervical musculature. E. Decreased ROM of the C-Spine w/ an uncompensated spinal curve

Correct: Answer D: The above Pt has degenerative joint disease of his C-Spine. The condition has been present for several years & has been getting progressively worse. When you examine this Pt you would expect to find chronic changes. There are some key phrases used to describe chronic SDs: cool, dry skin, absent/decreased edema or erythema, flaccid, ropy & fibrotic tissues, & pain that may be burning, achy or dull in nature. Answer A: Boggy & edematous paraspinal musculature is associated w/ acute SDs. In addition, severe cervical degenerative joint disease is usually associated w/ a decreased ROM. Answer B: Warm & moist skin texture w/ some erythema is associated w/ acute SDs. Answer C: Hypertonic paraspinals w/ severe, sharp pain upon palpation are associated w/ acute SDs. Answer E: An uncompensated spinal curve is associated w/ acute SDs. Chronic SDs are associated w/ compensated spinal curves. If a SD produces a spinal curve, over time other areas of the spine will tend to compensate w/ opposite spinal curves in attempt to straighten the body.

A 13 YO male comes to the ED w/ a R wrist & elbow pain following a fall. He states that he fell forward on his outstretched arm. On examination, there is no evidence of deformity of the forearm or wrist. Upon palpation he has exquisite pain between the extensor longus & abductor pollicis longus tenduons at the radiocarpal joint. There is mild edema at the radiocarpal joint. He has decreased flexion & extension at the wrist as well as decreased forearm supination. There is a tender point located at the musculotendonous insetion of the lateral epicondyle. X-Rays of the wrist & elbow are normal w/out evidence of fracture. Which of the following techniques is contraindicated in this Pt? A. ME to the upper extremity to increase supination B. Counterstrain to the tenderpoint C. Thoracic inlet releasee D. HVLA to the R wrist to improve ROM E. Posterior axillary fold technique

Correct: Answer D: The anatomic snuff box is located at the radiocarpal join & is bordered by the extensor pollicis longus & abductor pollicis longus tendons. Since the Pt has pain at the anatomic snuff box w/ some edema & decreased ROM at the wrist, he may have a fracture of the scaphoid bone. HVLa is contraindicated in areas of the fracture. Answer A: ME to the upper extremity to increase supination is not contraindicated because this technique can be modified so that no stress is applied to the radiocarpal joint. A cross extensor reflex ME could also be used. This would require treating the L upper extremity in order to achieve a therapeutic effect in the R upper extremity. Answer B: Counterstrain is not contraindicated because this technique can also be modified so that no stress is applied to the radiocarpal joint. Answer C & E: Thoracic inlet release & posterior axillary fold techniques are indicated in this scenario because both techniques will help open lymphatic channels & help decrease the edema in this Pt's R upper extremity.

Which of the following is true regarding the C-Spine? A. Hypertonic changes at the C4/C5 facet joint & degenerative changes at the corresponding joint of Luschka may cause C4 nerve root entrapment. B. The synovium of the uncovertebral joint is continous w/ that of the facet joint. C. The main motion of the OA joint is Rotation. D. The joints of Luschka play an important role in cervical Sidebending. E. Restriction w/in the sternocleidomastoid will decrease neck rotation to the controlateral side

Correct: Answer D: The joints of Luschka (also callled the uncovertebral joints) are only located in the Spine. Most authors agree that they are not true synovial joints, however they play an important role in cervical motion, especially Sidebending. They also serve to protect nerve roots from dis herniation. Answer A: Since the C5 nerve root exists between the C4 & C5 vertebrae, degenerative changes of the joint of Luschka & hypertrophy of the facet joint may cause C5 nerve root entrapment, not the C4 nerve root. Answer B: If the uncovertebral joint (joint of Luschka) was a true synovial joint (most authors think it is not) it's synovium would not be continous w/ that of the facet joint. This is because the facet joints & uncovertebral joints are located on opposite sides of the intervetebral foramen. Answer C: The main motion of the OA is flexion & extension. Approximately half of the extension & extension of the C-Spine stems from the OA joint. Answer E. Restriction w/in the SCM muscle will decrease Rotation to the ipsilateral side.

A 35 YO female who is obese complains of L foot pain. The pain is worse in the morning when she gets out of bed or if she stands at work for a long period of time. The pain is located on the bottom of her foot at the anterior-medial aspect of the calcaneus. The most likely Dx is: A. Pes planus B. Pes cavus C. Tarsal tunnel syndrome D. Plantar fasciitis E. Morton neuroma

Correct: Answer D: The most common cause of foot pain in outpatient medicine is plantar fasciitis, which results from constant strain on the plantar fascia at its insertion into the medial tubercle of the calcaneus. The most common causes are obesity, prolonged standing, & improper footwear. The pain is worse in the morning, but usually subsides after a few minutes of ambulation. Dx is confirmed by palpation over the plantar fascia on the medial heel. Answer A: Pes planus is a flattened longitudinal arch; it is not typically associated w/ pain. Answer B: Pes cavus is an exaggerated height of the longitudinal arch, it is also not typically associated w/ pain. Answer C: Tarsal tunnel syndrome is a complex of symptoms resulting from compression of the posterior tibial nerve or the plantar nerve in the tarsal tunner. Symptoms include pain & parasthesias of the sole of the foot. Answer E: Morton's neuroma is a fibroneuromatous rxn between the heads of the 3rd & 4th metatarsals. Pain is present at the forefoot at the site of the neuroma.

A 52 YO male w/ atrial fibrilation comes to your office complaining of a new onset R shoulder pain. The pain started 24 hours ago during his golf tournament. The Pt recalls that the pain came about immediately following a golf swing. The pain, which is the greatest at the tip of the R acromion does not radiate. He denies any numbness or tingling in his upper extremity. There is full passive ROM. At rest, w/ his arm by his side, the Pt has little pain. However, the pain becomes moderate to severe when he attempts to hold his arm above his head. X-Rays of his shoulder show no fracture. His medications include Coumadin & hydrochlorothiazide.

Correct: Answer D: The question describes a positive drop arm test. A rotator cuff tear is often associated w/ trauma & will result in sharp pain at the tip of the acromion, pain w/ active abduction & a positive drop arm test. Answer A: Restricted passive & active ROM is characteristic of adhesive capsulitis. Answer B: Bicipital tendonitis will result in pain at the bicipital groove & increased pain w/ shoulder & elbow flexion. Answer C: Osteoarthritis is characterized by a gradual onset of pain & limited passive ROM Answer E: Thoracic outlet syndrome is due to compression of the neurovascular bundle. This typically results in pain radiating into the upper extremity.

In a Pt w/ an anterior fibular head SD, which of the following statements is true? A. The above type of dysfunction does not easily respond to Tx B. The SD will increase the amount of passive plantar flexion C. It can result in foot drop D. The tibio-fibular synovial articulation will resist posterior glide E. It is often seen in common ankle sprains

Correct: Answer D: There are 2 tibio-fibular articulations. The proximal articulation is synovial & the distal articulation is syndesmotic. In an anterior fibular head dysfunction, the proximal tibio-fibular articulation will be displaced anteriorly & will resist posterior glide. Answer A: A fibular head SD responds well to manipulative Tx's. HVLA, ME & indirect methods are commonly used techniques. Answer B: A fibular head anterior is associated w/ an externally rotated talus causing the foot to appear more dorsiflexed & everted. Increased motion in these planes is seen as a result. Answer C: The common peroneal nerve courses posteriorly to the fibular head. Posterior fibular head dysfunction can result in compression of this nerve causing foot drop. Answer E: The most common ankle sprain occurs w/ the foot in supination (plantar flexion, inversion & internal rotation). These ankle sprains are associated w/ posterior fibular head SD's.

A Pt w/ L shoulder pain has the following findings: The L clavicle at the sternum is more cephalad than the R. The R sternoclavicular head of the clavicle moves inferiorly when the Pt shrugs his shoulders while the L does not. What is the most likely Dx? A. L anterior Rotation SD of the SC joint B. L anterior SD of the SC joint C. L inferior SD of the SC joint D. L superior SD of the SC joint E. L posterior SD of the SC joint

Correct: Answer D:The clavicle moves in 3 different directions. First it moves superior & inferior w/ shrugging & depressing of the shoulder. Second, it moves anterior & posterior w/ retraction & protraction of the shoulder. Third, it rotates anteriorly & posteriorly w/ internal & external rotation of the arm when it is abducted at 90°. Shrugging the shoulder will cause the lateral end of the clavicle to move superior; this will cause the medial end to move inferior. If it does not, this indicates that a SD is present. Since you name SD's in direction of freer motion, the clavicle has a superior SD at the sternum.

A 68 YO man is brought to your office for postural evaluation. The sagittal plane evaluation reveals that the Pt's entire body leans anteriorly away from a plumb line that hangs from the ceiling to the floor in the exam room. The Pt's weight is supported mainly by the metatarsals of his feet. Your postural assessment of this Pt would be: A. Military posture B. Swayback posture C. Posterior postural deviation D. Flat back posture E. Anterior postural deviation.

Correct: Answer E: Anterior postural deviation as viewed from the sagittal plane (in relation to a plumb line) is described as: entire body leans forward, deviating anteriorly from plumb line, Pt's weight is supported by metatarsals. Answer A: Military posture, as viewed from the sagittal plane (in relation to a plumb line)is described as: head tilted slightly posteriorly, cervical curve & thoracic curve normal, anterior cervical & posterior thoracic deviation from plumb line, anterior pelvic tilt, knees extended, ankles plantar flexed. Answer B: Swayback posture as viewed from the sagittal plane (in relation to a plumb line) is described as: head forward, C-Spine lordotic, T-Spine kyphotic, decreased lordosis of L-Spine, posterior tilt of pelvis, hip & knee joionts hyperextended. Answer C: Posterior postural deviations as viewed from the sagittal plane (in relation to a plumb line) is described as: entire body leans backward, deviating posteriorly, balance maintained by anterior thrust of pelvis & hips, & marked lordosis from mid-thoracic spine down. Answer D: Flat back posture as viewed from the sagittal plane (in relation to a plumb line) is described as: head forward, C-Spine has slightly increased lordosis, T-Spine slightly kyphotic in upper portion then flattens in lower segments, lumbar lordosis flattened, & hips & knees extended.

A 21 YO male is in the ICU after a MVA. He has sustained bilateral tibia/fibula fractures & several rib fractures. He underwent closed reduction/external fixation of his tib/fib fractures 24 hours ago. He has been extubated since the surgery & is complaining of severe pain at the surgical site as well as at the chest wall. Ultrasound of the lower extremities is negative for DVT. Which one of the following is the best statement regarding osteopathic Tx for this Pt? A. This Pt should receive osteopathic manipulation daily until he is transferred out of the unit. B. This Pt should receive direct active forms of Tx such as ME C. This Pt should receive 1 extensive session of osteopathic Tx then followed up in 2 weeks D. Osteopathic Tx is contraindicated in this Pt due to the acute nature of the illness E. This Pt should receive indirect myofascial release & diaphragm release to improve lymphatic return.

Correct: Answer E: Diaphragm release techniques are not contraindicated in this Pt. In addition, this Pt could also tolerate some indirect myofascial release techniques. Answer A & B: Due to the severity of this person's illness he would not be able to tolerate daily OMT, nor would he be able to tolerate direct active forms of OMT. Answer C: Osteopathic Tx for those individuals in the ICU should be limited in dosage. Usually these Pt's cannot tolerate extensive osteopathic Tx sessions. Answer D: Although this Pt is in critical condition he would be able to tolerate some mild forms of OMT.

You approach your osteopathic professor one day w/ a very serious question concerning premature ejaculation. You ask if there are any specific OMT techniques that may help this type of ejaculatory dysfunction. The professor explains that OMT may help alter visceral input form facilitated segments. Based on your knowledge of the autonomic system & visceral innervation, you come to the conclusion that tx of which structures may help the problem? A. Occipito-atlantal joint B. sacrum C. T4-T6 D. T8-T10 E. T12-L2

Correct: Answer E: Ejaculation is a sympathetically mediated response. This event is activated by T11-L2. OMT to this area may help a Pt who is having premature ejaculation. Answer B: Manipulation to the sacrum may help parasympathetic event (erection).

A 34 YO female comes into your office complaining of mild L-sided thoracic pain. The pain started about a week after the Pt began driving her new sports car that has very low riding seats. The pain is non-radiating & worsens w/ inhalation. X-Rays & EKG reveal no abnormalities. Your structural exam reveals that ribs 3 -5 on the L are more caudad & lag behind during inhalation. Tenderpoints are noted on ribs 3 - 5 in the L mid-axillary line. Which of the following most correctly describes the SD? A. Type I thoracic dysfunction B. Type II thoracic dysfunction C. R Sidebending thoracic dysfunction D. L rib 3 - 5 inhalation dysfunction E. L rib 3 - 5 exhalation dysfunction

Correct: Answer E: Exhalation dysfunctions are characterized by one or a group of ribs held down. These ribs typically lag behind in inhalation. Pain usually increases w/ inhalation. Also anterior rib tenderpoints (in the mid-axillary line) are often associated w/ anteriorly depressed ribs. Answer A, B & C: Although most ribs dysfunctions are due to thoracic dysfunctions, the case does not include a thoracic dysfunction, therefore it cannot be assumed that the Pt has one. Answer D: Inhalation dysfunctions are characterized by one or a group of ribs held up. These ribs typically lag behind in exhalation. Pain usually increases w/ exhalation. Also, posterior rib tenderpoints are often associated w/ posteriorly depressed ribs.

You are consulted to see a severely debilitated 87 YO male w/ complaints of mid-thoracic pain. He was in the ICU for 3 weeks & was transferred to a medical/surgical bed yesterday. He has a Hx of coronary artery disease, CHF & prostate cancer w/ vertebral metastisas. His back pain is localized to the mid thoracic region & seems to be musculoskeletal in nature. Which osteopathic manipulative technique would be best suited to relieve this Pt's symptoms? A. ME Tx w/ the spine extended B. Thoraco-abdominal diaphragm release C. HVLA D. Pedal (Dalrymple) pump E. Direct myofascial release

Correct: Answer E: Hospitalized Pt's typically respond better /w indirect techniques or gentle direct techniques. Often these types of Pts cannot w/stand aggressive Tx. There are a variety of direct myofascial techniques that a physician can do that will not interfere w/ this fragile state. Answer A: ME (regardless of the position of the spine) is contraindicated in Pts w/ low vitatlity such as Pts in the ICU Answer B & D: Techniques geared toward improving lymphatic return are relatively contraindicated in Pts w/ advanced stages of cancer. In addition, the thoracoabdominal diaphragm release & pedal pump are less likely to decrease this Pt's mid-thoracic pain since it seems to be musculoskeletal in nature & not directly related to lymphatic congestion. Answer C: HVLA is contraindicated in Pt's w/ bone metastasis because this may cause a pathologic fracture to the spine

Which of the following is the best statement regarding anatomy & biomechanics of the L-Spine? A. The L-Spine moves easiest about a sagittal axis B. Lumbarization increased the flexibility of the lumbosacral unit thus decreasing the likelihood of degenerative joint disease. C. The S1 nerve root will exit between the L5 & S1 vertebrae. D. Central disc herniations are more common than posterior-lateral disc herniations. E. Radicular symptoms from a herniated disc will often worsen when the L-Spine is flexed

Correct: Answer E: Lumbar flexion will push the nucleus pulposus posterior & stretch the nerve roots. This will usually worsen radicular symptoms. Answer A: The L-Spine moves easiest through flexion & extension. This is motion about a transverse axis & in a sagittal plane. Answer B: Lumbarization will alter the structure-function relationship of the lumbosacral junction, leading to early disc degeneration. Answer C: In the thoracic & lumbar regions the nerve root will exit the intervetebral foramina below it's corresponding segment. For example, the L5 nerve root will exit between L5 & S1. Answer D: The posterior longitudinal ligament narrows as it approaches the sacrum. This results in a posterio-lateral weakness making disc herniations more common in this area. Large central disc herniations that impinge S2-S4 can result in cauda equina syndrome, but this is uncommon.

A 17 YO female complains of anterior knee pain that gets worse when she runs, jumps or climbs stairs. PE reveals pain is reproducible on squatting & when the knee is flexed. There is point tenderness on the undersurface of the patella & there is some patellar crepitus. The most likely Dx is: A. Osgood-Shlatter's disease B. Baker's cyst C. Patellar tendonitis D. Housemaid's knee E. Patello-femoral syndrome

Correct: Answer E: Patello-femoral syndrome (AKA lateral femoral patella tracking syndrome) is among the most common causes of knee complaints in PC medicine, particularly among adolescents & young adults. The Pt's CC is anterior knee pain that is exacerbated w/ running, jumping & climbing stairs. Answer A & D: Osgood-Schlatter's is a traction apophysitis of the tibial tubercle. Pain & inflammation are located at the tibial tubercle. Housemaid's knee also has enlargement & pain at the tibial tuberosity, which is caused by constant kneeling on hard surfaces. Both these conditions are responsible for anterior knee pain, but at a very localized area, & do not account for point tenderness on the posterior aspect of the patella or patellar crepitus. Answer B: Baker's cyst is usuall:y found in the popliteal fossa of the knee & does not result in anterior knee pain. These cysts can be associated w/ degeneration of the posterior horn of the medial meniscus. Answer C: Patellar tendonitis is an inflammation of the patellar tendon usually caused by overuse. It results in pain at the insertion into the patella. Patellar crepitus is not an associated finding.

A Pt complains of bloody diarrhea. A colonoscopy reveals the mucosa at the recto-sigmoid region is edematous, friable, & w/ numerous erosions. At which spinal level would you expect to find palpatory changes related to viscerosomatic reflex? A. C3 - T5 B. T1 - T4 C. T5 - T9 D. T10 - T11 E. T12 - L2

Correct: Answer E: Pathologyin the recto-sigmoid area would cause a viscerosomatic change at the T12 - L2 spinal level. Answer A: C3 - C5 spinal segments are not involved in a viscerosomatic reflex in the recto sigmoid region. Answer B: T1 - T4 receives sympathetic innervation from the head & neck Answer C: T5 - T9 receives sympathetic innervation from the upper GI tract Answer D: T10 - T12 receives sympathetic innervation from the middle GI tract

Which of the following is true regarding goals of manipulation in a Pt w/ asthma? A. Bronchodilate by stimulating the parasympathetic nervous system B. Normalize the phrenic nerve w/ a CV4 technique C. Encourage perfusion of lung parenchyma by stimulating the sympathetic nervous system D. Decrease the thoraco-abdominal pressure gradient by redoming the diaphragm. E. Improve ventilation & perfusion (V/Q) function of the lungs w/ rib raising

Correct: Answer E: Rib raising will increase costal wall motion thus increasing ventilation. It will also normalize sympathetic tone thus increasing tissue perfusion. Answer A: Stimulating the sympathetic nervous system would cause bronchodilation; stimulating the parasympathetic nervous system would cause bronchorestriction. Answer B: Cranial manipulation may normalize the vagus nerve, not the phrenic nerve. Answer C: Decreasing the sympathetic nervous system will encourage perfusion of the lung parenchyma. Answer D: Redoming the diaphragm will increase the thoraco-abdominal pressure gradient & improve lymphatic return.

A 17 YO high school gymnast comes to your office for a High school physical. Throughout her school career she has been having intermittent back pain. The back pain is localized to the R thoraco-lumbar region & worsens w/ spinal extension. On examination, her R shoulder is lower than her L, & her R iliac crest is higher. Her R leg is 1.2 cm shorter than the left. L2 is flexed, Rotated right & Sidebent right. Her L-Spine appears to be sidebent to the R & L5 is Rotated left. She has full ROM & reverses her scoliotic curve on spinal motion testing. Pelvis shifts easily to the L & resistance is present w/ R pelvic shift. There is a tenderpoint in the center of her L buttock. On standing postural X-ray of the spine & pelvis the scoliotic curve measures 140 & the L femoral head is 12mm caudad compared to the R. A spina bifida occulta is noted on X-rays w/out evidence of spondylolisthesis. Which of the following most accurately describes this Pt's scoliosis? A. A functional dextroscoliosis B. A structural dextroscoliosis C. A structural levoscoliosis D. A structural kyphoscoliosis E. A functional levoscoliosis

Correct: Answer E: The L-Spine is sidebent to the right, therefore the Pt has a levoscoliosis. Since the Pt is able to reverse her scoliosis on ROM testing she has a functional scoliotic curve. Answer A & B: A dextroscoliosis describes a spine that is sidebent to the L. Answer B, C & D: A structural curve is a spinal curve that is relatively fixed & inflexible. A structural curve will not correct w/ sidebending in the opposite direction. Answer D: A kyphoscoliosis is a scoliosis that has resulted in an increase in kyphosis.

A 19 YO volleyball player dislocated her shoulder while spiking a volleyball several weeks ago. She immediately was taken to the ER where it was relocated. She was started in a rehab program working w/ her trainer & team physician. Since the injury, her pain has diminished however she still has some persistent weakness w/ arm abduction. On examination, strength testing is 5/5 except for arm abduction, which is 2/5. Muscle stretch reflexes in the upper extremities are normal & symmetric. X-Rays are normal. What is the most likely etiology of her persistent weakness? A. Injury to the dorsal scapular nerve B. Injury to the inferior gleno-humeral ligament. C. Complete tear of the supraspinatus tendon D. Incomplete rupture of the deltoid at its insertion site E. It is often seen in common ankle sprains

Correct: Answer E: The axillary nerve courses around the posterior aspect of the humeral head. The anterior/inferior dislocation can stretch the axillary nerve thus injuring it. Approximately 10 - 18% of shoulder dislocations will result in injury to the axillary nerve. Typical muscle stretch reflexes (deep tendon reflexes) that are tested in upper extremities are biceps, triceps, brachioradialis reflexes. Injury to the axillary nerve will not affect these reflexes. Answer A: The dorsal scapular nerve innervates the rhomboids & is not susceptible to injury w/ shoulder dislocation Answer B: Although anterior/inferior shoulder dislocations typically disrupt the inferior glenohumeral ligament, it is not the most likely cause of arm abduction weakness. Answer C: Although complete tears of the supraspinatus tendon may lead to arm abduction weakness, shoulder dislocations do not typically result in complete supraspinatus tears. Answer D: Shoulder dislocations are not likely to rupture the deltoid muscle

What area of the thoracic vertebra is most commonly palpated for evaluation of a Pt's thoracic SD? A. Spinous process B. Superior costal facet C. Pedicle D. Transverse costal facet E. Transverse processes

Correct: Answer E: The most accepted area of thoracic vertebra for palpatory evaluation of the thoracic spine is the TP Answer A: The SP could be used, but convention dictates that it is the TP Answer B, C & D: would be inneffective for Dx & are not really palpable.

A 54 YO alcoholic male Pt complains of tender bumps at the distal palmar crease of his hand. PE reveals subcutaneous nodules & palmar fascia contracture of the ring finger. The most likely Dx is? A. DeQuervain's disease B. Heberden node C. Syphilis D. Bouchard's node E. Dupuytren's contracture

Correct: Answer E: This Pt has Dupuytren's contracture. This condition is characterized by contracture of the palmar fascia & nodule formation. There appears to be a genetic predisposition & is frequently found in alcoholics. Answer A: DeQuervain's disease is a tendonitis of the abductor pollicis longus, extensor pollicic longus or extensor pollicis brevis tendons Answer B: Herberden's nodes are cartilaginous & bony enlargements of the distal interphalangeal joints of the fingers in degenerative joint diseases. It is most commonly seen in osteoarthritis. Answer C: Secondary stage of syphillis presents w/ a rash on the palms of the hands & feet, however, this Pt has nodules, not a rash. Answer D: Bouchard's nodes are cartilaginous & bony enlargements of the proximal interphalangeal joints of the fingers in degenerative joint diseases. It is most commonly seen in RA.

A Pt complains of CP, especially w hen she exhales. The pain is localized to ribs 8-10 on the L. These ribs are characterized as: A. Bifid ribs B. Atypical ribs C. True ribs D. Floating ribs E. False ribs

Correct: Answer E: ribs 8 - 10 are classified as false ribs. False ribs are those ribs that do not directly attach to the sternum. Answer A: Bifid ribs occur in 1-2% of the population, but this abnormality is present in 8.4% of Samoans. The condition is usually unilateral, & appears as an 8th true rib. Answer B:The atypical ribs are 1, 2, 11, 12. Sometimes ribs 10 is considered atypical. Answer C: True ribs attach to the sternum. The true ribs are 1 - 7 Answer D: The floating ribs are 11 & 12

Match the description with the best term: A Pt has a severely painful bunion in his R foot that results in a short stance phase on the R & a rapidly executed swing phase of the L leg. A. High steppage gait B. Ataxic gait C. Shuffling (festinating) gait D. Waddling gait E. Hemiplegic gait F. Elevated pelvis gait G. Antalgic gait H. Gluteus medius gait I. Scissor gait

Correct: Answer G: Antalgic gait is characterized by a short stance phase while the Pt is standing on the painful extremity. During this short stance phase there is a rapidly executed swing phase of the opposite leg. The Pt tries to avoid standing on a painful extremity.

Match the description with the best term: A 56 YO male is observed to shift his body to the R side when he walks. The Pt has a positive trendelenberg sign when standing on the R foot. A. High steppage gait B. Ataxic gait C. Shuffling (festinating) gait D. Waddling gait E. Hemiplegic gait F. Elevated pelvis gait G. Antalgic gait H. Gluteus medius gait I. Scissor gait

Correct: Answer H: This gait is characterized by a shift of the body toward the side of the weak gluteus medius. In this case the person's R gluteus medius is weak. He compensates by shifting his body weight toward the weak side. By doing this he prevents the L side of his pelvis from dropping.

Match the description with the best term: A 16 YO female w/ spastic diplegic cerebral palsy walks w/ her legs adducted, crossing alternately in front of one another. A. High steppage gait B. Ataxic gait C. Shuffling (festinating) gait D. Waddling gait E. Hemiplegic gait F. Elevated pelvis gait G. Antalgic gait H. Gluteus medius gait I. Scissor gait

Correct: Answer I: In scissor gait the legs are adducted, crossing alternatively in front of one another. Both adductors typically have spasticity. This is a common type of gait in children w/ cerebral palsy.

A 68 YO male complains of a HA & blurred vision for several months. PE reveals an abnormal opthalmotonometry. At what spinal levels would you expect to find the tissue texture changes related to a viscerosomatic reflex? A. T1-T3 B. T5-T6 C. T7-T9 D. T10-T12 E. T12-L2

Correct: Answer: This Pt most likely has glaucoma, & would have viscerosomatic changes at the T1-T4 spinal level. Answer B: T5-T6 spinal level is associated w/ the viscerosomatic reflex from the lungs & the esophagus. Answer C: T7-T9 spinal level is associated w/ the viscerosomatic reflex from the upper GI tract. Answer D: T10-T12 spinal level is associated w/ the viscerosomatic reflex from the middle GI tract & the gonads Answer E: T12-L2 spinal level is associated w/ the lower GI tract.


संबंधित स्टडी सेट्स

Chapter 35: Caring for Clients with HIV/AIDS

View Set

MGMT 4953 - Test 3 Campbell (Ch. 9 - 11)

View Set

EXAM 4 : antiinfective agents/antiinflammatory agents

View Set